Funcions - Toomates · Operacions amb funcions. Composició de funcions. Funció inversa. 2 Les...

168
FUNCIONS Llibre de text Gerard Romo Garrido

Transcript of Funcions - Toomates · Operacions amb funcions. Composició de funcions. Funció inversa. 2 Les...

Page 1: Funcions - Toomates · Operacions amb funcions. Composició de funcions. Funció inversa. 2 Les funcions de primer grau. → 3 Les funcions de segon grau. → 4 Les funcions polinòmiques.

FUNCIONS

Llibre de text

Gerard Romo Garrido

Page 2: Funcions - Toomates · Operacions amb funcions. Composició de funcions. Funció inversa. 2 Les funcions de primer grau. → 3 Les funcions de segon grau. → 4 Les funcions polinòmiques.

Toomates Coolección

Los documentos de Toomates son materiales digitales y gratuitos. Son digitales porque están pensados para ser consultados mediante un

ordenador, tablet o móvil. Son gratuitos porque se ofrecen a la comunidad educativa sin coste alguno. Los libros de texto pueden ser digitales o

en papel, gratuitos o en venta, y ninguna de estas opciones es necesariamente mejor o peor que las otras. Es más: Suele suceder que los mejores docentes son los que piden a sus alumnos la compra de un libro de texto en papel, esto es un hecho. Lo que no es aceptable, por inmoral y

mezquino, es el modelo de las llamadas "licencias digitales" con las que las editoriales pretenden cobrar a los estudiantes, una y otra vez, por

acceder a los mismos contenidos (unos contenidos que, además, son de una bajísima calidad). Este modelo de negocio es miserable, pues impide el compartir un mismo libro, incluso entre dos hermanos, pretende convertir a los estudiantes en un mercado cautivo, exige a los

estudiantes y a las escuelas costosísimas líneas de Internet, pretende pervertir el conocimiento, que es algo social, público, convirtiéndolo en un

producto de propiedad privada, accesible solo a aquellos que se lo puedan permitir, y solo de una manera encapsulada, fragmentada, impidiendo el derecho del alumno de poseer todo el libro, de acceder a todo el libro, de moverse libremente por todo el libro.

Nadie puede pretender ser neutral ante esto: Mirar para otro lado y aceptar el modelo de licencias digitales es admitir un mundo más injusto, es

participar en la denegación del acceso al conocimiento a aquellos que no disponen de medios económicos, en un mundo en el que las modernas tecnologías actuales permiten, por primera vez en la historia de la Humanidad, poder compartir el conocimiento sin coste alguno, con algo tan

simple como es un archivo "pdf". El conocimiento no es una mercancía.

El proyecto Toomates tiene como objetivo la promoción y difusión entre el profesorado y el colectivo de estudiantes de unos materiales didácticos libres, gratuitos y de calidad, que fuerce a las editoriales a competir ofreciendo alternativas de pago atractivas aumentando la calidad

de unos libros de texto que actualmente son muy mediocres, y no mediante retorcidas técnicas comerciales.

Este documento se comparte bajo una licencia “Creative Commons”: Se permite, se promueve y se fomenta cualquier uso, reproducción y edición de todos estos materiales siempre que sea sin ánimo de lucro y se cite su procedencia. Todos los documentos se ofrecen en dos

versiones: En formato “pdf” para una cómoda lectura y en el formato “doc” de MSWord para permitir y facilitar su edición y generar versiones

parcial o totalmente modificadas. Se agradecerá cualquier observación, comentario o colaboración a [email protected]

La biblioteca Toomates Coolección consta de los siguientes libros:

Metodología Problem-solving:

Geometría Axiomática GA pdf 1 2 ... 23 portada

Problemas de Geometría PG pdf 1 2 3 4 5 6 7

Introducción a la Geometría PI (en preparación) pdf doc

Teoría de números AR pdf 1 2 3

Trigonometría PT pdf doc

Desigualdades DE pdf doc

Números complejos PZ pdf doc

Álgebra PA pdf doc

Combinatoria PC pdf doc

Probabilidad PR pdf doc

Guía del estudiante de Olimpiadas Matemáticas OM pdf

Libros de texto (en catalán):

Àlgebra AG pdf 1 2

Funcions FU pdf doc

Geometria analítica GN pdf 1 2

Trigonometria TR

pdf doc

Nombres complexos CO pdf doc

Àlgebra Lineal 2n batxillerat AL pdf doc

Geometria Lineal 2n batxillerat GL pdf doc

Càlcul Infinitesimal 2n batxillerat CI pdf 1 2

Programació Lineal 2n batxillerat PL pdf doc

Recopilaciones de pruebas PAU España:

Catalunya TEC ST , Catalunya CCSS SC , Galicia SG

Recopilaciones de pruebas PAU Europa:

Portugal A SA, Portugal B SB

Recopilaciones de problemas olímpicos y preolímpicos:

IMO SI, OME SE, OMI SD, AIME SA , Cangur SR , Canguro SG , Kangourou SK ,

AMC12 (2008-2020) SM

Versión de este documento: 16/08/2020

Todos estos documentos se actualizan constantemente. ¡No utilices una versión anticuada! Descarga gratis la última versión de los documentos en los enlaces superiores.

www.toomates.net

Page 3: Funcions - Toomates · Operacions amb funcions. Composició de funcions. Funció inversa. 2 Les funcions de primer grau. → 3 Les funcions de segon grau. → 4 Les funcions polinòmiques.

Índex

1 Les funcions reals de variable real. → Operacions amb funcions. Composició de funcions. Funció inversa.

2 Les funcions de primer grau. →

3 Les funcions de segon grau. →

4 Les funcions polinòmiques. →

5 Les funcions de proporcionalitat inversa. →

6 Les funcions racionals. →

7 Les funcions radicals. →

8 Les funcions exponencials. →

9 Les funcions logarítmiques. →

10 Les funcions trigonomètriques. →

11 Les funcions definides a trossos. →

12 La funció valor absolut. →

Solucions. →

Page 4: Funcions - Toomates · Operacions amb funcions. Composició de funcions. Funció inversa. 2 Les funcions de primer grau. → 3 Les funcions de segon grau. → 4 Les funcions polinòmiques.

1 Funcions reals de variable real.

Definició de funció.

Una funció BAf : és una correspondència que assigna a cada element a de A un (i

només un) element del conjunt B, que denotem per mitjà de )(af . Es diu que )(af és

la imatge de a per f .

Si IRA i IRB direm que f és una funció real de variable real.

Formes de presentar una funció.

Un enunciat o regla ens informa amb paraules sobre la forma en què es relacionen

entre elles les variables. Per exemple: «El nombre de subscriptors a la nostra revista

creix uniformement des de l'any 2005..."

Una taula associa a determinats valors d’una de les variables els corresponents valors

de l’altra. Per exemple:

Una gràfica representa de forma clara i visual la relació que hi ha entre les variables.

Una fórmula o expressió algebraica, quan això és possible, la dependència exacta

que es dona entre variables; és a dir, com mitjançant unes determinades operacions

realitzades sobre una d’elles, en podem obtenir l’altra. Per exemple:

4193521)( xxN

Page 5: Funcions - Toomates · Operacions amb funcions. Composició de funcions. Funció inversa. 2 Les funcions de primer grau. → 3 Les funcions de segon grau. → 4 Les funcions polinòmiques.

Domini i recorregut.

El conjunt A on té sentit la funció, s'anomena domini de f . Es representa per fDom .

Per exemple, 2x no pertany al domini de la funció xxf )( perquè no existeix el

nombre 2)2( f .

Direm que un nombre b és imatge d'una funció f quan )(xfb per a cert x del

domini.

Per exemple: 21 és imatge de xxxf 4)( 2 perquè 21747)7( 2 f .

El recorregut d'una funció és el conjunt de totes les imatges. Es representa per fIm .

També s'anomena conjunt imatge. És difícil veure quin és el recorregut a partir de

l'equació d'una funció. El millor és fer-ho a partir de la gràfica i projectar-la sobre l'eix

y. El recorregut serà l'interval de l'eix y que queda pintat per la gràfica. El recorregut de

la funció és el conjunt de valors que adopta la funció.

La gràfica d’una funció ens permet veure, en un mateix dibuix, el domini de la funció

(eix d’abscisses, X) i les imatges (eix d’ordenades, Y ). Ens dóna molta informació

sobre el comportament de la funció en tot el seu domini.

Característiques singulars de la gràfica d'una funció.

Simetries.

Direm que una funció f és parella si )()( xfxf .

Direm que una funció f és imparella o senar si )()( xfxf

Periodicitat.

Direm que una funció f és periòdica de període 0T si la seva gràfica es va

repetint en intervals de longitud T.

Page 6: Funcions - Toomates · Operacions amb funcions. Composició de funcions. Funció inversa. 2 Les funcions de primer grau. → 3 Les funcions de segon grau. → 4 Les funcions polinòmiques.

Punts de tall.

Punts de tall amb l'eix X.

Són els valors del domini per als què 0)( xf

Punt de tall amb l'eix Y.

És el valor (si existeix) de )0(f .

Punts de tall entre dues gràfiques.

Són les possibles solucions de l'equació )()( xgxf

Page 7: Funcions - Toomates · Operacions amb funcions. Composició de funcions. Funció inversa. 2 Les funcions de primer grau. → 3 Les funcions de segon grau. → 4 Les funcions polinòmiques.

1.1.1

Donada la següent gràfica:

a) Domini de definició.

b) Imatge de la funció.

c) Punts de tall amb l'eix X.

d) Punt de tall amb l'eix Y.

e) Determina )4(f .

f) Resol 1)( xf .

1.1.2

Donada la següent gràfica d'una funció, determina:

a) )2(f , )0(f , )2(f , )3(f

b) El domini i el recorregut de la funció.

c) Punts de tall amb l'eix X (aproximadament)

d) Punt de tall amb l'eix Y.

e) Els valors de x per als què 3)( xf .

f) El mínim i el màxim de la funció.

Page 8: Funcions - Toomates · Operacions amb funcions. Composició de funcions. Funció inversa. 2 Les funcions de primer grau. → 3 Les funcions de segon grau. → 4 Les funcions polinòmiques.

1.1.3

Donada la següent gràfica d'una funció, determina:

a) )4(f , )2(f , )0(f , )4(f .

b) El domini i el recorregut de la funció.

c) Punts de tall amb l'eix X (aproximadament).

d) Punt de tall amb l'eix Y.

e) El mínim i el màxim de la funció.

1.1.4

Donades les següents gràfiques, determina:

a) )0(f , )3(g .

b) El domini i el recorregut de )(xf .

c) El domini i el recorregut de )(xg .

d) Punts de tall de )(xg amb l'eix X.

e) Punts de tall entre )(xf i )(xg .

Page 9: Funcions - Toomates · Operacions amb funcions. Composició de funcions. Funció inversa. 2 Les funcions de primer grau. → 3 Les funcions de segon grau. → 4 Les funcions polinòmiques.

1.1.5

Donades les següents gràfiques, determina:

a) )3(f , )3(g , )4(g , )4(f .

b) El domini i el recorregut de )(xf .

c) Punts de tall entre )(xf i )(xg .

Page 10: Funcions - Toomates · Operacions amb funcions. Composició de funcions. Funció inversa. 2 Les funcions de primer grau. → 3 Les funcions de segon grau. → 4 Les funcions polinòmiques.

1.1.6 Anàlisi d'una gràfica d'un diari.

Font: El Punt-Avui 11/04/2012

Espanya i Alemanya necessiten emetre bons a deu anys de forma regular per finançar-se. Si la rendibilitat

dels bons espanyols a deu anys és del 6,099%, i la dels bons alemanys del 2,621%, la diferència és del

3,478%, és a dir, de 347 punts bàsics. En aquest cas, la prima de risc espanyola seria de 347 punts. La

prima de risc dels països de la Unió Europea es calcula respecte de Alemanya perquè se suposa que el seu

deute públic és el que menor risc de impagament té, fins i tot se sol acceptar que el seu risc d'impagament

és 0.

1. Construeix una gràfica amb aquestes dades, però amb l’eix horitzontal complet, és a

dir, on apareguin també tots els dies que falten a la gràfica del diari.

2. Respon a les següents preguntes:

a) En quins dies s’ha observat una prima de risc per sobre de 350 punts?

b) En quins dies s’ha observat una prima de risc per sota dels 345 punts?

c) En quins dies s’ha observat una prima de risc entre 355 i 270 punts?

d) Quina ha sigut la variació de la prima de risc entre el 3 i el 10 d’Abril?

e) Determina els intervals on observem un comportament creixent i un comportament

decreixent de la prima de risc.

f) Determina les dies on s’han localitzat mínims i màxims. Quin és el mínim absolut i el

màxim absolut? En quins dies s’han localitzat?

Page 11: Funcions - Toomates · Operacions amb funcions. Composició de funcions. Funció inversa. 2 Les funcions de primer grau. → 3 Les funcions de segon grau. → 4 Les funcions polinòmiques.

1.2 Operacions amb funcions.

Suma, resta, multiplicació i divisió de funcions.

Les funcions es poden sumar, restar, multiplicar i dividir, operant les expressions

corresponents.

Suma de funcions: )()())(( xgxfxgf

Resta de funcions: )()())(( xgxfxgf

Multiplicació de funcions: )()())(( xgxfxgf

Divisió de funcions: )(

)()(

xg

xfx

g

f

si 0)( xg

Domini de definició de les operacions amb funcions.

Observem que l'únic problema afegit de domini el trobarem amb la divisió, perquè no

estarà definida quan el denominador sigui zero.

Exercici resolt.

Donades 1)( 2 xxf i 53)( xxg , determina

a) )1(gf b) )3( gf c) )5)(( gf d) 0

g

f

Solució:

a)

1082)1()1()1(8)1(

2)1(

gfgf

g

f

b)

14)4(10)3()3()1(4)3(

10)3(

gfgf

g

f

c)

5202026)5()5()5(20)5(

26)5(

gfgf

g

f

d)

5

1

)0(

)0(0

5)0(

1)0(

g

f

g

f

g

f

Page 12: Funcions - Toomates · Operacions amb funcions. Composició de funcions. Funció inversa. 2 Les funcions de primer grau. → 3 Les funcions de segon grau. → 4 Les funcions polinòmiques.

Exercici resolt.

Donades 98)( xxf i 12)( xxg , determina l'expressió algèbrica de les

següents funcions i el seu domini:

a) )(xgf b) )(xgf c) ))(( xgf d) xg

f

Solució:

a) 1298)()()( xxxgxfxgf

b) 1298)()()( xxxgxfxgf

c) 1298)()())(( xxxgxfxgf

d) 12

98

)(

)(

x

x

xg

xfx

g

f

Domini: el domini de 12)( xxg és ,2/1 , i com que la suma, resta i producte

de funcions no altera el domini, el domini de )(xgf , )(xgf , i ))(( xgf serà

aquest mateix: ,2/1 .

Ara bé, la funció )(

)(

xg

xfx

g

f

tampoc estarà definida si el denominador és zero:

2

101212)(0 xxxxg .

Per tant, el domini de )(

)(

xg

xfx

g

f

és ,2/1 .

1.2.1 Exercici.

Donades les gràfiques de )(xf i )(xg :

determina:

a) )4)(( gf b) )0(fg c) )1)(( gf d) )0(

f

g

e) )0(

g

f

Page 13: Funcions - Toomates · Operacions amb funcions. Composició de funcions. Funció inversa. 2 Les funcions de primer grau. → 3 Les funcions de segon grau. → 4 Les funcions polinòmiques.

1.2.2 Exercici.

Donada la següent taula de valors X-Y:

determina:

a) )4)(( gf b) )2( gf c) )1)(( gf d) )0(

g

f

Page 14: Funcions - Toomates · Operacions amb funcions. Composició de funcions. Funció inversa. 2 Les funcions de primer grau. → 3 Les funcions de segon grau. → 4 Les funcions polinòmiques.

1.3 Composició de funcions.

Definició de composició de funcions.

Les funcions es poden composar, és a dir, encadenar la sortida d'una a l'entrada d'una

altra. Per exemple:

9129523)()(5)(

23)(22

2

xxxxfgxfg

xxg

xxf

L'expressió )(xfg es llegeix g composta amb f de x.

- Recorda sempre que per avaluar )(xfg es comença per la funció de la

dreta, perquè és la primera que actua sobre la variable x.

- No has de confondre el símbol de composició " " amb el multiplicació "·"

Si interpretem les funcions com màquines que tenen una entrada i una sortida:

La composició de funcions és enganxar màquines en cadena: La sortida d'una a l'entrada

de l'altra:

Podem enganxar tantes màquines (funcions) com vulguessim,una darrera l'altra:

Page 15: Funcions - Toomates · Operacions amb funcions. Composició de funcions. Funció inversa. 2 Les funcions de primer grau. → 3 Les funcions de segon grau. → 4 Les funcions polinòmiques.

Sobre la no commutativitat de la composició de funcions.

Observa que la composició de funcions en general no és commutativa:

)()( xgfxfg

Per exemple: 5)(,23)( 2 xxgxxf

9129523)()( 22 xxxxfgxfg

1732153253)()( 222 xxxxgfxgf

En la vida real, el cas més conegut de no commutativitat de funcions és el fer-se un got

de Colacao. El resultat no és el mateix si aboques primer la llet i després el Cola-Cao

que si primer aboques el cola-Cao al got buit i després la llet, perquè et queda tot ple de

grumets. Tanmateix, això no passa amb el Nesquik. Ni passa amb la llet calenta. No hi

ha commutativitat només amb Cola-cao i amb llet freda.

f

g

gf

fg

Page 16: Funcions - Toomates · Operacions amb funcions. Composició de funcions. Funció inversa. 2 Les funcions de primer grau. → 3 Les funcions de segon grau. → 4 Les funcions polinòmiques.

Domini de la composició de funcions.

Per determinar domini de fg hem de trobar els valors de x que compleixen que:

- x és en el domini de f

- )(xf és en el domini de g.

Exercici resolt.

Donades 12)( xxf i 1

4)(

xxg , determina

a) )2(gf b) )3(fg

Solució:

a) 72)2(7142)4(41

4

12

4)2(

gfgffg

b) 2

13)3(

2

1

8

4

17

4)7(71)3(2)3(

fgfggf

Molta atenció amb el tatxar operacions!

Si 2)( xxf i xxg )( , llavors 2)( xxgf no és x , xxgf )(

xxxfg 2)( , i ,0fgDom

Page 17: Funcions - Toomates · Operacions amb funcions. Composició de funcions. Funció inversa. 2 Les funcions de primer grau. → 3 Les funcions de segon grau. → 4 Les funcions polinòmiques.

Exercici resolt.

Amb les funcions 14)( xxf i xxxg 52)( 2 , comprova que, en general,

)()( xgfxfg :

Solució:

7363252021632520)1816(2

)14(5)14(2)14())(()(

222

2

xxxxxxxx

xxxgxfgxfg

12081)52(4)52())(()( 222 xxxxxxfxgfxgf

I clarament : 120873632 22 xxxx

1.3.1

Donades les funcions xxxf 2)( 2 i xxg 3)( , resol les següents equacions:

a) 0)( xgf b) 05)( 2 xxfg

1.3.2

Donades 9)( 2 xxf i 52)( xxg , resol la inequació 0xgf

Page 18: Funcions - Toomates · Operacions amb funcions. Composició de funcions. Funció inversa. 2 Les funcions de primer grau. → 3 Les funcions de segon grau. → 4 Les funcions polinòmiques.

1.4 Composició de funcions i translacions.

Donat un nombre 0k

Translació vertical cap a dalt:

kxgxf )()(

Translació vertical cap a baix:

kxgxf )()(

Translació horitzontal cap a l'esquerra:

)()( kxgxf

Translació horitzontal cap a la dreta:

)()( kxgxf

...i la composició de les dues anteriors: qkxgxf )()(

Page 19: Funcions - Toomates · Operacions amb funcions. Composició de funcions. Funció inversa. 2 Les funcions de primer grau. → 3 Les funcions de segon grau. → 4 Les funcions polinòmiques.

1.5 Composició de funcions: Dilatacions i contraccions.

Donat un nombre 0k

Dilatació vertical: 1k

)()( xgkxf

Contracció vertical 1k

)()( xgkxf

Contracció horitzontal 1k

)()( xkgxf

Dilatació horitzontal 1k

)()( xkgxf

La composició de funcions i la seva interpretació geomètrica és clau per a

resoldre problemes algèbric complicats i no ofegar-nos en un mar d'àlgebra.

Estudia, per exemple, l'exercici 8.3.5.

Page 20: Funcions - Toomates · Operacions amb funcions. Composició de funcions. Funció inversa. 2 Les funcions de primer grau. → 3 Les funcions de segon grau. → 4 Les funcions polinòmiques.

1.5 Composició de funcions: Simetries.

Simetria horitzontal:

)()( xgxf

Simetria vertical:

)()( xgxf

Simetria central:

)()( xgxf

Page 21: Funcions - Toomates · Operacions amb funcions. Composició de funcions. Funció inversa. 2 Les funcions de primer grau. → 3 Les funcions de segon grau. → 4 Les funcions polinòmiques.

1.6 La funció inversa.

Definició de funció inversa.

Donada una funció f , anomenem inversa de f, i escriurem 1f , a qualsevol funció tal

que

idffff 11

On id és la funció identitat: xxid )( , la funció "que no fa res".

Amb la interpretació de màquines de l'apartat 1.3, donada una màquina f , volem trobar

una màquina 1f que desfaci allò que fa f :

Només les funcions bijectives tenen inversa.

Amb les funcions, "elevar a a la menys ú" no és dividir.

Amb els nombres, a

a11 , per exemple: 25.0

4

14 1

Però amb les funcions, "elevar a la menys ú" es fer la inversa, no dividir.

1.6.1

Donades les funcions

83)( xxf 3

8)(

xxg

Determina gf i fg . Què observes?

Inversa d'una composició de funcions.

Si f es posar-me la sabata, 1f és treure'm la sabata.

Si q es posar-me el mitjó, 1q és treure'm el mitjó.

Llavors gf és primer posar-me el mitjó i després, a sobre, posar-me la sabata.

La seva inversa és 11 fg , és a dir, primer treure'm la sabata i després treure'm el

mitjó.

El que no faré és 11 gf , és a dir, intentar treure'm el mitjò sense treure'm la sabata

primer (comprova-ho!)

Amb aquesta idea al cap, la inversa de la composició de funcions és la composició

d'inverses, però invertint també l'ordre:

111 fggf

Page 22: Funcions - Toomates · Operacions amb funcions. Composició de funcions. Funció inversa. 2 Les funcions de primer grau. → 3 Les funcions de segon grau. → 4 Les funcions polinòmiques.

Càlcul de la funció inversa.

1. Comprovarem si la funció és bijectiva.

La manera més fàcil de veure si una funció és bijectiva és fer-ho gràficament:si per

qualsevol línia horitzontal que tracem sobre la gràfica aquesta talla la funció només en

un punt, la funció és injectiva. Si el recorregut són tots els reals, la funció és exhaustiva.

I si es donen aquests dos supòsits, la funció és bijectiva.

2. Aïllarem la y en funció de x.

3. Canviarem la x per la y a l'equació associada.

Exemple resolt.

Calcula la funció inversa de la funció 2)( 3 xxf .

Solució:

1. Comprovem que és una funció bijectiva observant la seva gràfica

2. Aïllem la y en funció de x:

xyxyxyxxfy 3333 2222)(

3. Canviarem la x per la y a l'equació associada:

31 2)( xxfy

Page 23: Funcions - Toomates · Operacions amb funcions. Composició de funcions. Funció inversa. 2 Les funcions de primer grau. → 3 Les funcions de segon grau. → 4 Les funcions polinòmiques.

Interpretació gràfica de la funció inversa.

La gràfica d'una funció i la seva inversa són simètriques respecte la bisectriu del 1r i 3r

quadrant, és a dir, respecte la recta xy .

Amb l'exemple anterior:

Page 24: Funcions - Toomates · Operacions amb funcions. Composició de funcions. Funció inversa. 2 Les funcions de primer grau. → 3 Les funcions de segon grau. → 4 Les funcions polinòmiques.

2 Les funcions de primer grau.

2.1 Característiques de la funció de primer grau.

Ara és un bon moment per repassar...

Les equacions de primer grau. (Tema 8 del Llibre d'Àlgebra)

Les inequacions de primer grau (Apartat 15.1 del Llibre d'Àlgebra)

Sistemes d'equacions lineals amb dues incògnites (Tema 16 del Llibre d'Àlgebra)

La funció de primer grau apareix també quan estudiem...

Les rectes en el pla (Tema 2 del Llibre de Geometria)

La funció de primer grau.

Una funció es diu que és polinòmica de primer grau, si és del tipus:

baxxfy )(

on a i b són nombres reals i 0a .

Gràfica d'una funció de primer grau.

La seva gràfica sempre és una recta amb pendent a que talla l'Eix Y al punt ),0( b

Page 25: Funcions - Toomates · Operacions amb funcions. Composició de funcions. Funció inversa. 2 Les funcions de primer grau. → 3 Les funcions de segon grau. → 4 Les funcions polinòmiques.

2.2 De l'expressió algèbrica a la gràfica.

Exercici resolt.

Representa gràficament la funció 73)( xxf .

Solució:

Fem una taula de valors X-Y i els representem en un sistema de coordenades:

Les gràfiques de les funcions de primer grau sempre son rectes, i per tant només

cal fer una taula X-Y amb dos valors

Exemple resolt.

Representa gràficament la funció 12

3)( xxfy

4122

3)2(2

1102

3)0(0

fyx

fyx

Representem gràficament els punts )1,0( i )4,2( els unim amb una recta:

Page 26: Funcions - Toomates · Operacions amb funcions. Composició de funcions. Funció inversa. 2 Les funcions de primer grau. → 3 Les funcions de segon grau. → 4 Les funcions polinòmiques.

2.2.1

Representa gràficament les següents funcions.

a) 25

4)( xxf b) 2

4

5)( xxf

c) 44

7)( xxf d) 5

2

5)( xxf

e) 44

1)( xxf f) 4)( xxf

Page 27: Funcions - Toomates · Operacions amb funcions. Composició de funcions. Funció inversa. 2 Les funcions de primer grau. → 3 Les funcions de segon grau. → 4 Les funcions polinòmiques.

2.3 De la gràfica a l'expressió algèbrica.

Funció de primer grau coneixent dos punts de la seva gràfica.

Exemple resolt.

Determina la funció de primer grau que passa pels punts ( 1, 2 ) i ( 6 , 4).

Solució:

La funció serà de la forma baxxf )( per a certs a i b que hem de trobar.

Substituïm els valors X-Y de l'enunciat:

baba

baba

6464)4,6(

212)2,1(

Resolem el sistema d'equacions que hem obtingut:

5

8

5

222

5

252642642

64

2

64

2

ab

aaaaaaab

ab

ba

ba

La funció és 5

8

5

2)( xxf

Comprovem el resultat:

25

10

5

8

5

2)1()2,1( f

45

20

5

8

5

12

5

86

5

2)6()4,6( f

Page 28: Funcions - Toomates · Operacions amb funcions. Composició de funcions. Funció inversa. 2 Les funcions de primer grau. → 3 Les funcions de segon grau. → 4 Les funcions polinòmiques.

2.3.1

Els costos d’edició, en euros, de x exemplars d’un llibre venen donats per y=21x+24

(x>0). Quant costa editar 8 exemplars? I 80 exemplars? Escriu la funció que

proporciona el cost per exemplar. Per molts exemplars que es publiquin, quin és el cost

unitari com a mínim?

Page 29: Funcions - Toomates · Operacions amb funcions. Composició de funcions. Funció inversa. 2 Les funcions de primer grau. → 3 Les funcions de segon grau. → 4 Les funcions polinòmiques.

2.4 Intersecció de gràfiques.

Trobar els punts de tall entre dues gràfiques )(xf i )(xg equival a resoldre l'equació

)()( xgxf

Exercici resolt.

Determina els punts d'intersecció de les gràfiques associades a les funcions

3)( xxf i 42)( xxg

Solució:

x

xx

xx

xgxf

1

243

423

)()(

Busquem punts, per tant, hem de trobar també la "y":

231)1(1 fx

El punt de tall és )2,1(

Fem la comprovació:

231)1( f

)1(2424)1(2)1( fg

2.4.1

A cada apartat, determina el punt de tall entre les dues gràfiques:

a) 34)( xxf i 15)( xxg

b) 12)( xxf i 23)( xxg

c) 12)( xxf i 43)( xxg

d) 34)( xxf i 25)( xxg

Page 30: Funcions - Toomates · Operacions amb funcions. Composició de funcions. Funció inversa. 2 Les funcions de primer grau. → 3 Les funcions de segon grau. → 4 Les funcions polinòmiques.

3 Les funcions de segon grau.

Ara és un bon moment per repassar...

Les equacions de segon grau. (Tema 11 del Llibre d'Àlgebra)

Les funcions de segon grau apareixen també quan estudiem...

Les paràboles (Tema 4 del Llibre de Geometria)

3.1 Característiques de la funció de segon grau.

La funció de segon grau.

Una funció es diu que és polinòmica de segon grau, si és del tipus:

cbxaxxfy 2)(

on a, b, i c són nombres qualssevol, i a mai és zero.

Gràfica de la funció de segon grau.

La gràfica de la funció de segon grau és sempre una paràbola.

Vèrtex de la paràbola.

Anomenem vèrtex de la paràbola, a l’únic punt de la paràbola que està sobre l’eix de

simetria. Coincideix amb el punt més alt o més baix respecte l’eix de les y’s.

El vèrtex té per coordenades:

a

bf

a

b

2,

2

Eix de simetria de la paràbola.

Anomenem eix de simetria de la paràbola a la recta (vertical) que al “doblegar” el full

faria que els dues “branques” coincidissin.

L’eix de simetria, és la recta vertical d’equació:a

bx

2

Page 31: Funcions - Toomates · Operacions amb funcions. Composició de funcions. Funció inversa. 2 Les funcions de primer grau. → 3 Les funcions de segon grau. → 4 Les funcions polinòmiques.

3.2 De l'expressió algèbrica a la gràfica. 3.2.1

Les funcions de segon grau x2 i –x

2.

Representa gràficament les funcions 2)( xxfy i 2)( xxgy

x 2)( xxfy

-4 164)4(2 fy

-3

-2

-1

0

1

2

3

4

Page 32: Funcions - Toomates · Operacions amb funcions. Composició de funcions. Funció inversa. 2 Les funcions de primer grau. → 3 Les funcions de segon grau. → 4 Les funcions polinòmiques.

x 2)( xxgy

-4 164)4( 2 gy

-3

-2

-1

0

1

2

3

4

Completa la següent taula :

Funció 2)( xxfy 2)( xxgy

Domini

Recorregut

Punts de tall amb l’eix X

Punts de tall amb l’eix Y

Simetria

Creixement i decreixement

Màxims i mínims

Curvatura

Page 33: Funcions - Toomates · Operacions amb funcions. Composició de funcions. Funció inversa. 2 Les funcions de primer grau. → 3 Les funcions de segon grau. → 4 Les funcions polinòmiques.

Exercici resolt

Representa gràficament la funció 2)( 2 xxfy

Solució:

Fem una taula de valors X-Y:

i els passem a un sistema de coordenades:

Page 34: Funcions - Toomates · Operacions amb funcions. Composició de funcions. Funció inversa. 2 Les funcions de primer grau. → 3 Les funcions de segon grau. → 4 Les funcions polinòmiques.

3.2.2

Representa gràficament la funció 6)( 2 xxxfy

x 6)( 2 xxxfy

-4 146)4()4()4( 2 fy

-3

-2

-1

0

1

2

3

4

Completa la següent taula :

Funció 6)( 2 xxxfy

Domini

Recorregut

Punts de tall amb l’eix X

Punts de tall amb l’eix Y

Simetria

Creixement i decreixement

Màxims i mínims

Curvatura

Page 35: Funcions - Toomates · Operacions amb funcions. Composició de funcions. Funció inversa. 2 Les funcions de primer grau. → 3 Les funcions de segon grau. → 4 Les funcions polinòmiques.

3.2.3

Representa gràficament la funció 2)( 2 xxxfy

x 2)( 2 xxxfy

-4 182)4()4()4( 2 fy

-3

-2

-1

0

1

2

3

4

Completa la següent taula :

Funció 6)( 2 xxxfy

Domini

Recorregut

Punts de tall amb l’eix X

Punts de tall amb l’eix Y

Simetria

Creixement i decreixement

Màxims i mínims

Curvatura

Page 36: Funcions - Toomates · Operacions amb funcions. Composició de funcions. Funció inversa. 2 Les funcions de primer grau. → 3 Les funcions de segon grau. → 4 Les funcions polinòmiques.

3.2.4

Punts notables de la gràfica de segon grau.

a) Donada la funció 14)( 2 xxxf . Determina:

- Punt de tall amb l’eix Y.

- Punts de tall amb l’eix X.

- Vèrtex.

- Eix de simetria.

Representa els elements anteriors en un sistema X-Y i amb la seva ajuda traça la gràfica

de la funció.

Fes ara el mateix amb les següents funcions:

b) 342)( 2 xxxf

c) 15)( 2 xxxf

d) 103)( 2 xxxf

Page 37: Funcions - Toomates · Operacions amb funcions. Composició de funcions. Funció inversa. 2 Les funcions de primer grau. → 3 Les funcions de segon grau. → 4 Les funcions polinòmiques.

Punts de tall amb l'eix X.

Determinar els punts de tall amb l'eix X equival a resoldre una equació de segon grau.

Ho hem d'intentar per factorització, i hem d'intentar factoritzar la funció mentalment. Si

no, aplicarem la fórmula.

Exercici resolt.

Determina els punts de tall amb l'eix X de la funció xxxf 5)( 2 .

Solució:

Hem de resoldre l'equació 052 xx

550 2 xxxx Traiem factor comú x

505

050

xx

xxx Apliquem el Principi del producte nul.

Els punts de tall són 0x i 5x

Page 38: Funcions - Toomates · Operacions amb funcions. Composició de funcions. Funció inversa. 2 Les funcions de primer grau. → 3 Les funcions de segon grau. → 4 Les funcions polinòmiques.

3.3 Funcions de segon en forma estàndard.

Tota funció de segon grau es pot escriure de la forma

khxaxf 2)()(

I en aquest cas, l'eix de simetria és la recta vertical hx i el vèrtex de la funció és el

punt ),( kh .

Si 0a la gràfica estarà oberta cap amunt, i si 0a , la gràfica estarà oberta cap avall.

La gràfica de la funció khxaxf 2)()( és la gràfica de la funció 2)( axxg ,

sotmesa a una translació horitzontal h unitats a la dreta i una translació vertical de k

unitats.

Exemple:

Representa gràficament la funció 8)3(2)( 2 xxf .

Partim de la gràfica de la funció 22)( xxg , que té les branques cap avall:

Punts de tall amb els eixos.

Si la funció ve expressada en forma estàndard, els punts de tall amb l'eix X es poden

determinar sense necessitat de resoldre cap equació de segon grau.

Per exemple, amb la funció anterior 8)3(2)( 2 xxf

123

422243

42

8)3(

8)3(2

08)3(2)(

2

2

2

xx

xxx

x

x

xxf

Page 39: Funcions - Toomates · Operacions amb funcions. Composició de funcions. Funció inversa. 2 Les funcions de primer grau. → 3 Les funcions de segon grau. → 4 Les funcions polinòmiques.

3.3.1

Donada la funció

1)3()( 2 xxf

Determina el seu eix de simetria, el vèrtex, els punts de tall amb els eixos i representa-la

gràficament.

3.3.2

Associa cada funció amb la seva corresponent gràfica.

1)1()( 2 xxf 1)1()( 2 xxg 1)1()( 2 xxh 1)1()( 2 xxj

3.3.3

Determina les coordenades del vèrtex de la gràfica de cada funció:

a) 1)3(2)( 2 xxf b) 12)2(3)( 2 xxf c) 5)1(2)( 2 xxf

d) 8)4(2)( 2 xxf e) 382)( 2 xxxf f) 1123)( 2 xxxf

g) 82)( 2 xxxf h) 182)( 2 xxxf

3.3.4

A cadascuna de les funcions següents, i sense representar-la gràficament, dedueix si té

un màxim o un mínim i quines són les seves coordenades:

a) 1123)( 2 xxxf b) 382)( 2 xxxf c) 384)( 2 xxxf

d) 3122)( 2 xxxf e) xxxf 55)( 2 f) xxxf 66)( 2

Page 40: Funcions - Toomates · Operacions amb funcions. Composició de funcions. Funció inversa. 2 Les funcions de primer grau. → 3 Les funcions de segon grau. → 4 Les funcions polinòmiques.

3.3.5

Estudi de funcions de segon grau.

a) Escriu la seva forma estàndard.

b) Determina el seu vèrtex V, els punts de tall amb l'eix X i el punt de tall amb l'eix Y.

c) Dibuixa la seva gràfica.

d) Determina el domini i el recorregut.

a) 32)( 2 xxxf b) xxxf 6)( 2 c) xxxf 63)( 2

d) 46)( 2 xxxf e) 342)( 2 xxxf f) 1124)( 2 xxxf

3.3.6

Determina la funció de segon grau coneguts el seu vèrtex i un punt de la seva gràfica.

Dóna el resultat en forma estàndard.

a) Vèrtex ( 0 , 0 ) i passa pel punt ( -2 , 8 ).

b) Vètex ( 2 , 0 ) i passa pel punt (1 , 3).

c) Vèrtex ( -3 , 0 ) i passa pel punt ( -5 , -4 ).

d) Vèrtex ( 0 , 1 ) i passa pel punt ( -1 , 0 ).

e) Vèrtex ( 2 , 5 ) i passa pel punt ( 3 , 7 ).

f) Vèrtex ( -3 , 4 ) i passa pel punt ( 0 , 0 ).

3.3.7

Passa cadascuna de les següents funcions a forma estàndard aplicant el mètode de

completar quadrats. Un cop passada a forma estàndard, determina el seu vèrtex.

a) xxxf 4)( 2 b) 22)( 2 xxxf c) 2106)( xxxf

d) 238)( xxxf e) 982)( 2 xxxf

Page 41: Funcions - Toomates · Operacions amb funcions. Composició de funcions. Funció inversa. 2 Les funcions de primer grau. → 3 Les funcions de segon grau. → 4 Les funcions polinòmiques.

3.4 De la gràfica a l'expressió algebraica.

Ja vam veure a l'apartat anterior que tota funció de primer grau baxxf )( queda

determinada per dos punts de la seva gràfica, i que necessitem resoldre un sistema de

dues equacions i dues incògnites.

Una funció de segon grau cbxaxxf 2)( queda determinada per tres punts de la

seva gràfica. La seva determinació ens obliga a resoldre un sistema de tres equacions

amb tres incògnites, però veurem amb el següent exemple que és més fàcil del que

sembla, que té truco.

Exercici resolt.

Determina la funció de segon grau cbxaxxf 2)( sabent que

5)2( f , 8)1( f , 4)6( f , és a dir, que passa pels punts )5,2( , )8,1( i )4,6( .

Solució:

Substituïm els valors donats a l'expressió algebraica:

cbacbaff 24)2()2()2(55)2( 2

cbacbaff 11)1(88)1( 2

cbacbaff 63666)6(44)6( 2

Per tant, hem obtingut el següent sistema de tres equacions i tres incògnites:

cba

cba

cba

6364

8

245

Però, observa que la part " c " desapareix! Per exemple, restant la segona equació a la

primera:

bacba

cba333

8

245

Page 42: Funcions - Toomates · Operacions amb funcions. Composició de funcions. Funció inversa. 2 Les funcions de primer grau. → 3 Les funcions de segon grau. → 4 Les funcions polinòmiques.

I restant la tercera equació a la segona:

bacba

cba5354

6364

8

I obtenim un sistema de dues equacions amb dues incògnites:

ba

ba

5354

333

Que podem resoldre, per exemple, per substitució:

ba

bababa

5354

11333

40

314031535354535355)1(354 bbbbbbbb

40

9

40

31

40

40

40

3111

ba

Un cop resolt el sistema de dues equacions, determinem la "c" amb qualsevol de les tres

equacions inicials:

20

149

40

298

40

31294405

40

312

40

945

40

312

40

945245

c

ccba

Així doncs, la nostra funció és

20

149

40

31

40

9)( 2 xxxf

Comprovem que ho hem fet bé:

520

149)2(

40

31)2(

40

9)2( 2 f

820

1491

40

311

40

9)1( 2 f

420

1496

40

316

40

9)6( 2 f

3.4.1

A cada apartat, determina la funció de segon grau que passa pels següents punts:

a) ( 1, 6 ) , ( 3 , 20 ) , ( -2 , 15 )

b) ( 2, 7 ) , ( -1 , -2 ) , ( 4 , 3)

c) ( 1, -1 ) , ( -1 , 9 ) , ( 3 , 13)

Page 43: Funcions - Toomates · Operacions amb funcions. Composició de funcions. Funció inversa. 2 Les funcions de primer grau. → 3 Les funcions de segon grau. → 4 Les funcions polinòmiques.

3.5 Intersecció de gràfiques.

Trobar els punts de tall entre dues gràfiques )(xf i )(xg equival a resoldre l'equació

)()( xgxf

Primer grau vs. Segon grau.

Exercici resolt.

Determina els punts d'intersecció de les gràfiques associades a les funcions

56)( 2 xxxf i 5)( xxg

Solució:

01070556556)()( 222 xxxxxxxxxgxf

(Recorda: sempre hem d'intentar factoritzar abans que tirar de fórmula i sempre

intentem factoritzar de cap)

Busquem dos nombres que el seu producte sigui 10 i la seva suma -7... provant...

provant... Ja està! -5 i -2

Per tant

202

5050)2)(5(1072

xx

xxxxxx

Busquem punts, per tant, hem de trobar també les "y":

055)5(5 gx

352)2(2 gx

Comprovem que, efectivament, són punts de tall:

)5(0530255565)5(5 2 gfx

)2(351245262)2(2 2 gfx

Els punts de tall són )0,5( i )3,2(

Page 44: Funcions - Toomates · Operacions amb funcions. Composició de funcions. Funció inversa. 2 Les funcions de primer grau. → 3 Les funcions de segon grau. → 4 Les funcions polinòmiques.

3.5.1

Determina els punts d'intersecció de les gràfiques associades a les funcions

a) 54)( 2 xxxf i 13)( xxg

b) 32)( 2 xxxf i 73)( xxg

c) 54)( 2 xxxf i 32)( xxg

(Recorda: Resol les equacions factoritzant, sense fer servir la fórmula"menys b més

menys arrel...")

Gràfiques tangents.

Direm que dues gràfiques són tangents quan tenen un únic punt de tall. A aquest punt

de tall únic li direm "punt de tangència".

3.5.2

Determina els punts d'intersecció de les gràfiques associades a les funcions

128)( 2 xxxf i 132)( xxg

3.5.3

Determina els punts de tall entre 23)( 2 xxxf i 2)( xxg sense fer servir la

calculadora. Representa gràficament aquestes dues funcions i comprova que, efectivament, els

punts trobats són punts d'intersecció.

Page 45: Funcions - Toomates · Operacions amb funcions. Composició de funcions. Funció inversa. 2 Les funcions de primer grau. → 3 Les funcions de segon grau. → 4 Les funcions polinòmiques.

4 Les funcions polinòmiques.

Ara és un bon moment per repassar...

Polinomis i operacions amb polinomis. (Tema 9 del Llibre d'Àlgebra)

Factorització i equacions polinòmiques. (Tema 10 del Llibre d'Àlgebra)

4.1 Característiques de les funcions polinòmiques. Definició.

La seva expressió general es

01

1

1 ...)( axaxaxaxf n

n

n

n

, amb IRai

Les funcions polinòmiques no presenten cap tipus de problema de domini, i per tant el

seu domini sempre és IR.

La gràfica de les funcions polinòmiques.

Les gràfiques de les funcions polinòmiques són com filferros que es van doblegant

tantes vegades com el seu grau.

Atura't i pensa...

Tots els polinomis de grau senar tenen, almenys, una arrel real. Perquè?

Sabies que...?

Nessie, el monstre del llac Ness, té forma de cúbica (funció polinòmica de grau 3)

Page 46: Funcions - Toomates · Operacions amb funcions. Composició de funcions. Funció inversa. 2 Les funcions de primer grau. → 3 Les funcions de segon grau. → 4 Les funcions polinòmiques.

4.1.1

Determina quines gràfiques no corresponen a funcions polinòmiques:

4.1.2

De quin grau pot ser el polinomi que té associada la següent gràfica:

A) 2 B) 3 C) 4 D) 5

4.1.3

Observa la següent pauta:

El nombre màxim de regions formades connectant n punts en una circumferència és

241823624

1)( 234 nnnnnf

a) Quin és el grau d'aquest polinomi?

b) Quin és el seu coeficient principal?

c) Detemina el nombre màxim de regions formades connectant 5 punts en una

circumferència. Comprova el resultat dibuixant els punts i comptant a mà les regions.

d) Quants punts hauriem de dibuixar per a tenir 99 regions?

Page 47: Funcions - Toomates · Operacions amb funcions. Composició de funcions. Funció inversa. 2 Les funcions de primer grau. → 3 Les funcions de segon grau. → 4 Les funcions polinòmiques.

El criteri del coeficient principal.

Donat el polinomi

01

1

1 ...)( axaxaxaxf n

n

n

n

Si el grau del polinomi és senar:

Si el grau del polinomi es parell:

Page 48: Funcions - Toomates · Operacions amb funcions. Composició de funcions. Funció inversa. 2 Les funcions de primer grau. → 3 Les funcions de segon grau. → 4 Les funcions polinòmiques.

4.2 De l'expressió algèbrica a la gràfica.

Exercici resolt.

Representa gràficament la funció 235)( 23 xxxxf , mitjançant una taula de

valors X-Y.

Solució.

Completem una taula de valors X-Y

322)2(3)2(5)2()2(2 23 fx

72)1(3)1(5)1()1(1 23 fx

22)0(3)0(5)0()0(0 23 fx

...

Marquem aquests punts en un sistema de coordenades X-Y:

No passarem sempre tots els punts. Hem d'excloure aquells que siguin massa

grans. En aquest cas no representem (-2, -32) i (5, 17)

Ara només cal unir aquests punts, recordant que la gràfica d'una funció de tercer grau és

com Nessie, el monstre del llac Ness:

Page 49: Funcions - Toomates · Operacions amb funcions. Composició de funcions. Funció inversa. 2 Les funcions de primer grau. → 3 Les funcions de segon grau. → 4 Les funcions polinòmiques.

4.3 Punts notables d'una funció polinòmica.

Punts de tall amb l'eix X

Els punts de tall amb l'eix X són les solucions de l'equació 0)( xf .També

s'anomenen arrels del polinomi.

La clau per trobar les arrels d'un polinomi és la factorització, i si no es pot, Ruffini. I si

no es pot amb Ruffini, per computadora.

Exemple resolt.

Determina els punts de tall amb l'eix X de la funció xxxf 4)( 3 .

Solució.

Hem de resoldre l'equació 043 xx i ho fem factoritzant el polinomi:

440 23 xxxx Traiem factor comú x

)2(20 xxx Apliquem la identitat notable ))((22 bababa

202

202

0

)2(20

xx

xx

x

xxx Apliquem el principi del producte nul.

Els punts de tall amb l'eix X són: 0x , 2x i 2x

Page 50: Funcions - Toomates · Operacions amb funcions. Composició de funcions. Funció inversa. 2 Les funcions de primer grau. → 3 Les funcions de segon grau. → 4 Les funcions polinòmiques.

Exercici resolt.

Determina les arrels del polinomi 33)( 23 xxxxf

Solució:

Hem de trobar les solucions de l'equació 033 23 xxx .

033 23 xxx Hem de resoldre aquesta equació.

0)3(1)3(2 xxx Agrupem factors repetits.

0)1)(3( 2xx Tornem a agrupar factors repetits

101

3030)1)(3(

2

2

xx

xxxx Apliquem el Criteri del producte nul

Les solucions són 1,1,3 xxx

Exercici resolt.

Determina les arrels del polinomi 234 44)( xxxxf

Solució:

044 234 xxx Hem de resoldre aquesta equació.

04422 xxx Traiem factor comú 2x

0222 xx Factoritzem 442 xx (de cap, mentalment!)

202

0)2(

00

02 2

2

22

xx

x

xx

xx Apliquem el criteri del producte nul.

Les solucions són 0x i 2x :

Page 51: Funcions - Toomates · Operacions amb funcions. Composició de funcions. Funció inversa. 2 Les funcions de primer grau. → 3 Les funcions de segon grau. → 4 Les funcions polinòmiques.

Factorització amb el mètode de Ruffini.

4.3.1

Factoritza els següents polinomis (fent servir Mètode de Ruffini):

a) 602122 23 xxx

b) 123123 23 xxx

c) 246254182 234 xxxx

4.3.2

Determina els punts de tall i representa gràficament les següents funcions:

a) 9393)( 23 xxxxf

b) 181822)( 23 xxxxf

c) 6043213)( 234 xxxxxf

Page 52: Funcions - Toomates · Operacions amb funcions. Composició de funcions. Funció inversa. 2 Les funcions de primer grau. → 3 Les funcions de segon grau. → 4 Les funcions polinòmiques.

4.4 De la gràfica a l'expressió algèbrica.

4.4.1

Determina les funcions polinòmiques donat el seu grau i els seus punts de tall amb els

eixos. Escriu les funcions en forma de suma de potències d’x.

a) Grau del polinomi: 3 - Punt de tall amb l’eix Y: 8 - Punts de tall amb l’eix X: -2,1 i 2

-6 -4 -2 2 4 6

-20

-10

10

20

30

b) Grau del polinomi: 3 - Punt de tall amb l’eix Y: -9 - Punts de tall amb l’eix X: -3,1 i 3

-6 -4 -2 2 4 6

-40

-20

20

40

c) Grau del polinomi: 4 - Punt de tall amb l’eix Y: 24 - Punts de tall amb l’eix X: -3 , -2,

1 i 2

-4 -3 -2 -1 1 2 3

20

40

60

80

d) Grau del polinomi: 5 - Punt de tall amb l’eix Y: 45 - Punts de tall amb l’eix X: -5, -3,

-1, 1 i 3

-6 -4 -2 2 4

-400

-200

200

400

Page 53: Funcions - Toomates · Operacions amb funcions. Composició de funcions. Funció inversa. 2 Les funcions de primer grau. → 3 Les funcions de segon grau. → 4 Les funcions polinòmiques.

4.4.2

Aquí tens quatre gràfiques:

A B

C D

... i quatre expressions algèbriques:

i) xxxxf 93)( 23 ii) 128)( 23 xxxxf

iii) 3232)( xxxxf iv) 3299)( xxxxf

Asssocia cada gràfica amb la seva expressió algèbrica.

Page 54: Funcions - Toomates · Operacions amb funcions. Composició de funcions. Funció inversa. 2 Les funcions de primer grau. → 3 Les funcions de segon grau. → 4 Les funcions polinòmiques.

Problema resolt.

Si el punt )15,5( pertany a la gràfica de la funció polinòmica )1)(4()( xxaxxf

a) Determina el valor de a .

b) Determina les arrels del polinomi, és a dir, els seus punts de tall amb l'eix X.

c) Representa gràficament la funció.

Solució:

a) )15,5( pertany a la gràfica de la funció és equivalent a dir que

8

112015645)15)(15(5)5(15 aaaaf

b) Hem de resoldre l'equació 0)( xf

101

404

0

08/1

)1)(4(8

1)(0

xx

xx

x

mai

xxxxf

Les arrels són 0x , 4x , 1x .

c) La funció és de grau 3, una cúbica, amb coeficient principal positiu, que talla l'eix X

en 4,0,1 .

A més a més, 0)10)(40(08

1)0( f

Amb això ja tenim una visió aproximada de com és la seva gràfica:

4.4.3

Si el punt )3,2( pertany a la gràfica de la funció polinòmica 2)2)(3()( xxaxf

a) Determina el valor de a .

b) Determina les arrels del polinomi, és a dir, els seus punts de tall amb l'eix X.

c) Representa gràficament la funció.

Page 55: Funcions - Toomates · Operacions amb funcions. Composició de funcions. Funció inversa. 2 Les funcions de primer grau. → 3 Les funcions de segon grau. → 4 Les funcions polinòmiques.

4.5 Transformacions de funcions polinòmiques.

Transformacions de funcions cúbiques.

4.5.1

Sobre la base de la funció 3)( xxg , i els seus punts notables, representa gràficament

les següents funcions, sense fer cap taula de valors, fent servir el mètode de les

translacions:

a) 1)2()( 3 xxf b) 1)1()( 3 xxf c) 2)3()( 3 xxf

Page 56: Funcions - Toomates · Operacions amb funcions. Composició de funcions. Funció inversa. 2 Les funcions de primer grau. → 3 Les funcions de segon grau. → 4 Les funcions polinòmiques.

4.5.2

Sobre la base de la funció 3)( xxg , i els seus punts notables, representa gràficament

les següents funcions, sense fer cap taula de valors, fent servir el mètode de les

translacions:

a) 32)(3 xxf b) 42)(

3 xxf c) 23)(

3 xxf

Page 57: Funcions - Toomates · Operacions amb funcions. Composició de funcions. Funció inversa. 2 Les funcions de primer grau. → 3 Les funcions de segon grau. → 4 Les funcions polinòmiques.

4.10 Repàs de funcions polinòmiques (I).

4.10.1

Determina els punts de tall de la funció 43)( xxf amb els eixos.

4.10.2

Determina la funció de primer grau baxxf )( que passa pels punts )11,2(P i

)2,1(Q

4.10.3

Donada la funció de segon grau 45)( 2 xxxf , determina:

a) Punt de tall de la funció amb l’eix Y.

b) Punts de tall de la funció amb l’eix X.

c) Vèrtex de la paràbola.

d) Eix de simetria.

e) Representa gràficament la funció sense fer taula de valors, assenyalant els valors

anteriors:

Page 58: Funcions - Toomates · Operacions amb funcions. Composició de funcions. Funció inversa. 2 Les funcions de primer grau. → 3 Les funcions de segon grau. → 4 Les funcions polinòmiques.

4.10.4

Determina la funció de segon grau cbxaxxf 2)( que passa pels punts )5,1(P ,

)2,2(Q )2,0( R

4.10.5

Determina els punts de tall A i B entre la funció 23)( xxf i la funció

532)( 2 xxxg

4.10.6

a) Factoritza la funció 162812)( 234 xxxxxf

b) Determina el punt de tall de la funció amb l’eix Y.

c) Determina els punts de tall de la funció amb l’eix X.

4.10.7

Determina l’única funció polinòmica )(xf de tercer grau que té associada la següent

gràfica, si sabem que 18)1( f . Escriu la funció com a suma de potències d’x.

Page 59: Funcions - Toomates · Operacions amb funcions. Composició de funcions. Funció inversa. 2 Les funcions de primer grau. → 3 Les funcions de segon grau. → 4 Les funcions polinòmiques.

4.10.8

a)Escriu una funció polinòmica de grau igual o superior que no tingui cap punt de tall

amb l’eix X.

b) Escriu una funció polinòmica de grau igual o superior a 3 que només tall l’eix X en el

punt x = 4 .

4.10.9

Donada la funció 1)( 23 bxaxxxf , determina el valor dels paràmetres a i b si

sabem que 3)2( f i 9)1( f

Page 60: Funcions - Toomates · Operacions amb funcions. Composició de funcions. Funció inversa. 2 Les funcions de primer grau. → 3 Les funcions de segon grau. → 4 Les funcions polinòmiques.

4.11 Repàs de funcions polinòmiques (II).

4.11.1

Determina els punts de tall de la funció 12)( xxf amb els eixos.

4.11.2

Determina la funció de primer grau baxxf )( que passa pels punts )5,3( P i

)10,2(Q

4.11.3

Donada la funció de segon grau 142)( 2 xxxf , determina:

a) Punt de tall de la funció amb l’eix Y.

b) Punts de tall de la funció amb l’eix X.

c) Vèrtex de la paràbola.

d) Eix de simetria.

e) Representa gràficament la funció sense fer taula de valors, assenyalant els valors

anteriors:

Page 61: Funcions - Toomates · Operacions amb funcions. Composició de funcions. Funció inversa. 2 Les funcions de primer grau. → 3 Les funcions de segon grau. → 4 Les funcions polinòmiques.

4.11.4

Determina la funció de segon grau cbxaxxf 2)( que passa pels punts

)1,2( A , )2,1(B )1,0( C

4.11.5

Determina els punts de tall A i B entre la funció 23)( xxf i la funció

32)( 2 xxxg

4.11.6

Factoritza la funció 12496)( 234 xxxxxf

4.11.7

Donada la funció polinòmica 12142)( 3 xxxf

a) Determina el punt de tall de la funció amb l’eix Y.

b) Determina els punts de tall de la funció amb l’eix X.

4.11.8

Determina l’única funció polinòmica )(xf de grau 3 que talla l’eix X pels punts

(2,0) (-3,0) i (0,0) i a més a més compleix 8)1( f . Escriu aquesta funció com a suma

de potències d’x.

4.11.9

Donada la funció 13)( 2 axxxf , determina el valor del paràmetre a si sabem que

36)3( f

4.11.10

Resol l’equació 2

652

x

xx

Page 62: Funcions - Toomates · Operacions amb funcions. Composició de funcions. Funció inversa. 2 Les funcions de primer grau. → 3 Les funcions de segon grau. → 4 Les funcions polinòmiques.

4.12 Repàs de funcions polinòmiques (III).

4.12.1

Factoritza 15239)( 23 xxxxf

4.12.2

Resol l’equació 0455446 234 xxxx

4.12.3

Determina una funció polinòmica de grau 3 que talla l’eix X als punts 3, 5 i -2.

Representa aquesta funció com a suma de potències d’x.

4.12.4

Determina la funció polinòmica de grau 3 que talla l’eix X pels mateixos punts 3, 5 i -2

però que a més a més passa pel punt ( 2 , -36).

4.12.5

Determina una funció polinòmica de grau 4 que només tingui com a punts de tall amb

l’eix X els valors x = 2 i x = -3.

4.12.6

Determina una funció polinòmica de grau 4 que té la següent gràfica associada:

4.12.7

Determina el valor de k per a que la funció 142)( 23 xkxxxf compleixi

5)2( f

4.12.8

Determina els valors de k per a que la funció

2)1(2)( 2234 xkxxkxxf

passi pel punt ( 1 , -2 )

4.12.9

Determina els valors a i b per a que la funció 7)( 23 bxxaxxf compleixi

9)1( f i 11)2( f

Page 63: Funcions - Toomates · Operacions amb funcions. Composició de funcions. Funció inversa. 2 Les funcions de primer grau. → 3 Les funcions de segon grau. → 4 Les funcions polinòmiques.

4.13 Repàs de funcions polinòmiques (IV).

4.13.1

Amb l’ajuda de la calculadora científica, avalua la següent funció al valor x donat.

Escriu el resultat amb 2 decimals.

86.0

323)( 2

x

xxxf

4.13.2

Determina el vèrtex V=(x,y) de la gràfica de la funció 123)( 2 xxxf

4.13.3

Determina els punts A i B d’intersecció entre la paràbola f(x) i la recta g(x).

13)(

42)( 2

xxg

xxxf

4.13.4

Determina els punts de tall amb els eixos associats a la funció polinòmica de tercer grau.

4812123)( 23 xxxxf

4.13.5

Determina la funció f(x) = ax2 + bx + c que passa pels punts P, Q i R.

P = ( -3 , -16 ) Q = ( 0 , 2 ) R = ( 1 , 4 )

Page 64: Funcions - Toomates · Operacions amb funcions. Composició de funcions. Funció inversa. 2 Les funcions de primer grau. → 3 Les funcions de segon grau. → 4 Les funcions polinòmiques.

4.13.6

Determina els punts de tall entre les funcions p(x) i q(x)

426)( 23 xxxxp

2)( 2 xxxq

4.13.7

Determina la funció polinòmica de tercer grau f(x) donats els seus tres punts de tall amb

l’eix X i un valor de la seva taula. Escriu aquesta funció com a suma de potències d’x.

Punts de tall:{ 2 , -1 , -4 }, Punt de tall amb l’eix Y: 24

Page 65: Funcions - Toomates · Operacions amb funcions. Composició de funcions. Funció inversa. 2 Les funcions de primer grau. → 3 Les funcions de segon grau. → 4 Les funcions polinòmiques.

5 Les funcions de proporcionalitat inversa.

Les funcions de proporcionalitat inversa apareixen també quan estudiem...

Les hipèrboles (Tema 6 del Llibre de Geometria)

5.1 Característiques de les funcions de proporcionalitat inversa.

Funció de proporcionalitat inversa.

La funció de proporcionalitat inversa és la funció

x

kxf )( , amb 0k

Gràfica de la funció de proporcionalitat inversa.

La seva gràfica és una hipèrbola equilàtera, els eixos de la qual són els eixos de

coordenades.

Les característiques d’una funció racional són aquestes:

- El seu domini és tota la recta real menys el valor x = 0: 0 xfDom

- Té simetria respecte a l’origen de coordenades, o sigui, és una funció imparella.

- La funció és decreixent si k > 0, i les seves branques se situen en el primer i tercer

quadrant.

- La funció és creixent si k < 0, i les seves branques se situen en el segon i quart

quadrant.

- No té punts de tall amb els eixos, encara que la gràfica s’hi aproxima tant com

vulguem en prolongar-la indefinidament:

La funció "aterra" en l'eix X quan x es fa més i més gran.

La funció "s'enlaira" en l'eix Y quan x s'apropa a 0.

Però la gràfica mai arriba a tocar cap eix.

Page 66: Funcions - Toomates · Operacions amb funcions. Composició de funcions. Funció inversa. 2 Les funcions de primer grau. → 3 Les funcions de segon grau. → 4 Les funcions polinòmiques.

Exemple resolt.

Suposem que y varia de forma inversament proporcional a x, i que 3y si 8x .

Determina la funció representa-la gràficament.

Solució:

x

kxfy )( Escrivim el model de la funció de proporcionalitat inversa.

83

k Substituïm y per 3 i x per 8

k83 Aïllem la k

xy

24 Escrivim la k obtinguda a la funció.

Per representar-la, construïm i omplim una taula de valors X-Y:

Page 67: Funcions - Toomates · Operacions amb funcions. Composició de funcions. Funció inversa. 2 Les funcions de primer grau. → 3 Les funcions de segon grau. → 4 Les funcions polinòmiques.

5.1.1

Escriu l’equació de la funció que té per gràfica una hipèrbola com la de la figura amb el

centre de simetria desplaçat al punt (2,-1).

5.1.2

Una aixeta amb un cabal de 8 litres/min triga 42 minuts en omplir un dipòsit. Quant

trigaria si el cabal fos de 24 litres/min? Escriu la funció cabaltemps.

Exercici resolt.

La base b d'un paral·lelogram d'àrea fixa varia de forma inversament proporcional a la

seva l'altura h, i b = 12 cm si h = 8 cm. Determina b quan h = 3 cm.

Solució:

h

kb Escrivim la funció de proporcionalitat inversa amb lletres.

812

k Substituïm pels valors donats a l'enunciat.

96k Trobem la constant k resolent l'equació resultant.

hb

96 Ja tenim la fórmula de relació.

3

96b Substituïm el valor h donat

32b Calculem el resultat.

La base mesura 32 cm quan l'altura mesura 3 cm.

Page 68: Funcions - Toomates · Operacions amb funcions. Composició de funcions. Funció inversa. 2 Les funcions de primer grau. → 3 Les funcions de segon grau. → 4 Les funcions polinòmiques.

6 Les funcions racionals.

Ara és un bon moment per repassar...

Fraccions algebraiques i equacions racionals. (Tema 12 del Llibre d'Àlgebra)

Les funcions racionals apareixen també quan estudiem...

Funcions racionals sense derivació (Tema 2 del Llibre de Càlcul)

6.1 Característiques de les funcions racionals.

Definició.

La seva expressió general és )(

)()(

xq

xpxf on )(xp i )(xq són polinomis.

Domini d'una funció racional.

Les funcions racionals presenten problemes de domini quan s'anul·la el denominador:

0)( xqIRfDom

Exemples:

a) 1

56)(

2

x

xxxf 1 IRfDom

b) 3

9)(

2

x

xxf 3 IRfDom

c) 45

3)(

2

xx

xxf 4,1 IRfDom

Page 69: Funcions - Toomates · Operacions amb funcions. Composició de funcions. Funció inversa. 2 Les funcions de primer grau. → 3 Les funcions de segon grau. → 4 Les funcions polinòmiques.

Punts de tall amb l'eix X.

Determinar els punts de tall amb l'eix X equival a resoldre una equació racional

igualada a zero. El denominador passa multiplicant i se'n va, però pot donar problemes

(vegeu el segon exercici resolt).

Exercici resolt.

Determina els punts de tall amb l'eix X de la funció 5

32)(

2

x

xxxf .

Solució:

Hem de resoldre l'equació 05

32)(

2

x

xxxf

0)5(0322 xxx El denominador "passa" multiplicant a zero, bye bye!...

)1)(3(320 2 xxxx Queda el numerador, que hem de factoritzar, (o

aplicar la fórmula)

101

303)1)(3(0

xx

xxxx Apliquem el Principi del Producte Nul.

Les solucions són 3x i 1x . Les comprovem:

02

0

53

3323)3(3

2

fx

02

0

5)1(

3)1(2)1()1(1

2

fx

Page 70: Funcions - Toomates · Operacions amb funcions. Composició de funcions. Funció inversa. 2 Les funcions de primer grau. → 3 Les funcions de segon grau. → 4 Les funcions polinòmiques.

Exercici resolt.

Determina els punts de tall amb l'eix X de la funció 1

1)(

2

x

xxf .

Solució:

Hem de resoldre l'equació 01

1)(

2

x

xxf

0)1(01 2 xx El denominador "passa" multiplicant a zero, bye bye!...

1x Queda el numerador, que només té solució 1x

Però al comprovar aquesta solució veiem que queda fora del domini de definició de la

funció:

0

0

11

11)1(1

2

fx no està definit (és una discontinuïtat evitable)

Aquesta funció no té cap punt de tall amb l'eix X:

Page 71: Funcions - Toomates · Operacions amb funcions. Composició de funcions. Funció inversa. 2 Les funcions de primer grau. → 3 Les funcions de segon grau. → 4 Les funcions polinòmiques.

6.2 Simplificació de funcions racionals.

Exemple resolt.

Simplifica la funció 2

2)(

2

2

xx

xxxf

Solució:

Factoritzem numerador i denominador:

1)1)(2(

2

2

2)(

2

2

x

x

xx

xx

xx

xxxf si 2x

La funció no està definida per a 2x i per a 1x

Exemple resolt.

Simplifica la funció 4

4

123

2)(

2

x

x

x

xxf

Solució:

Factoritzem:

22

4

43

2

4

4

123

2)(

2

xx

x

x

x

x

x

x

xxf

I "tatxem" els factors repetits:

)2(3

1

2

1

3

1)(

xxxf si 2x i 4x

La funció no està definida per a 2x i per a 4x

Si tatxes factors, no oblidis mai d'afegir la coletilla: "La funció no està definida

per a..."

Page 72: Funcions - Toomates · Operacions amb funcions. Composició de funcions. Funció inversa. 2 Les funcions de primer grau. → 3 Les funcions de segon grau. → 4 Les funcions polinòmiques.

6.3 Asímptotes verticals.

Definició.

Direm que la recta vertical ax és una asímptota vertical de la gràfica de la funció

racional )(

)()(

xq

xpxf quan )(xf quan ax

És a dir, quan la funció )(xf es fa més i més gran sense límit quan x a mida que x

s'apropa més i més a ax .

Exemple.

Les rectes 3x i 3x són asímptotes verticals de 9

)(2

2

x

xxf

Mètode pràctic.

Les asímptotes verticals de )(

)()(

xq

xpxf les trobarem quan s'anul·la el denominador:

0)( xq

i no s'anul·la el numerador: 0)( xp .

Si s'anul·la numerador i denominador tenim una "indeterminació 0/0" i pot ser

una asímptota o no. S'han d'estudiar amb mètodes específics.

Page 73: Funcions - Toomates · Operacions amb funcions. Composició de funcions. Funció inversa. 2 Les funcions de primer grau. → 3 Les funcions de segon grau. → 4 Les funcions polinòmiques.

Exercici resolt.

Determina i representa gràficament les asímptotes de la funció 9

)(2

x

xxf

Solució:

Factoritzem numerador i denominador:

Observem que 9 és un quadrat i recordem la identitat notable: ))((22 bababa

)3)(3(9)(

2

xx

x

x

xxf

El denominador s'anul·la per 3x i 3x , i amb aquests valors no s'anul·la el

numerador, per tant no estem davant de cap indeterminació 0/0. Les asímptotes de la

funció són les rectes verticals 3x i 3x :

Page 74: Funcions - Toomates · Operacions amb funcions. Composició de funcions. Funció inversa. 2 Les funcions de primer grau. → 3 Les funcions de segon grau. → 4 Les funcions polinòmiques.

Indeterminació 0/0.

Determina i representa gràficament les asímptotes de la funció 9

3)(

2

x

xxf

Solució:

Factoritzem numerador i denominador:

)3)(3(

3

9

3)(

2

xx

x

x

xxf

El denominador s'anul·la per 3x i amb aquest valor no s'anul·la el numerador. Per

tant, aquesta funció té una asímptota vertical en 3x .

Però, Atenció! Per a 3x també s'anul·la el numerador, i per tant estem davant d'una

indeterminació 0/0:

0

0

60

0

)33)(33(

33)3(

f

Simplifiquem numerador i denominador "tatxant" el factor repetit 3x .

La funció resultant és

Aquesta funció no està definida per a 3x , però no hi ha asímptota. Hem resolt la

indeterminació.

Si el factor s'elimina, la asímptota se'n va, però a la gràfica li queda una petita

marca, un petit foradet, com si fos una espinilla mal curada en la cara. I no ens

hem d'oblidar de dibuixar-lo.

Page 75: Funcions - Toomates · Operacions amb funcions. Composició de funcions. Funció inversa. 2 Les funcions de primer grau. → 3 Les funcions de segon grau. → 4 Les funcions polinòmiques.

6.3.1 Decideix quina gràfica correspon a cada funció:

1) 1

1)(

xxf 2)

1

1)(

xxf 3)

x

xxf

1)(

4)

x

xxf

1)(

5) 1

1)(

x

xxf 6)

1

1)(

x

xxf

Page 76: Funcions - Toomates · Operacions amb funcions. Composició de funcions. Funció inversa. 2 Les funcions de primer grau. → 3 Les funcions de segon grau. → 4 Les funcions polinòmiques.

7 Les funcions radicals.

Ara és un bon moment per repassar...

Expressions i equacions amb radicals (Tema 13 del Llibre d'Àlgebra)

7.1 Característiques de les funcions radicals.

Definició.

Anomenem funció radical quan la variable independent x es troba davall el signe

radical. n xgxf )()(

Recorda:

La funció 2)( xxf no és bijectiva, la seva inversa xxf )(1 només està

definida en ,0 .

La funció 3)( xxf és bijectiva, la seva inversa 31 )( xxf està definida en

tot IR.

Memoritza les gràfiques de les funcions 2x , x , 3x , 3 x

Page 77: Funcions - Toomates · Operacions amb funcions. Composició de funcions. Funció inversa. 2 Les funcions de primer grau. → 3 Les funcions de segon grau. → 4 Les funcions polinòmiques.

7.2 De l'expressió algèbrica a la gràfica.

Exercici resolt.

Representa gràficament la funció arrel quadrada xxfy )(

Construeix una taula de valors X-Y. Observa que la arrel quadrada no està definida per

a nombres negatius.

Exercici resolt.

Representa gràficament la funció arrel quadrada 3 44)( xxfy

Construeix una taula de valors X-Y. Observa que la arrel cúbica de nombres negatius sí

està definida.

7.2.1

Representa gràficament la funció 1)( xxf

Page 78: Funcions - Toomates · Operacions amb funcions. Composició de funcions. Funció inversa. 2 Les funcions de primer grau. → 3 Les funcions de segon grau. → 4 Les funcions polinòmiques.

7.2.2

Representa gràficament la funció arrel quadrada: xxfy )( mitjançant la

calculadora científica:

7.2.3

Representa gràficament la funció arrel cúbica 3)( xxfy mitjançant la calculadora

científica:

Page 79: Funcions - Toomates · Operacions amb funcions. Composició de funcions. Funció inversa. 2 Les funcions de primer grau. → 3 Les funcions de segon grau. → 4 Les funcions polinòmiques.

7.3 Domini de definició de les funcions radicals.

Si l’índex n és parell, el domini és ,0

si l’índex n és imparell, el domini és tota la recta real.

Exercici resolt.

Calcula el domini de la funció 1)( xxf i representa-la.

Solució:

,1101 fDomxx

Representem la seva gràfica amb l'ajut d'una petita taula de valors:

x 1)( xxf

-1 0

0 1

1 41.12

2 73.13

7.3.1

Determina el domini de definició de les següents funcions:

a) 32)( xxf b) 16)( 2 xxf

c) )5)(3()( xxxf d) x

xf1

)(

e) 3

2)(

x

xxf f)

65

4)(

2

xx

xxf

g) 6)( 2 xxxf h) )ln()( xxf

i) )sin()( xxf

7.3.2 (Repàs)

Resol les següent equacions:

a) 71 xx b) xx 21113 c) xxx 4122

7.3.3 M

Determina el domini de la funció

4

2

3

1)(

x

x

x

xxf

Page 80: Funcions - Toomates · Operacions amb funcions. Composició de funcions. Funció inversa. 2 Les funcions de primer grau. → 3 Les funcions de segon grau. → 4 Les funcions polinòmiques.

7.4 Punts notables de la gràfica de funcions amb arrels quadrades.

7.4.1

a) Determina:

- L’extrem P de la gràfica.

- El punt de tall Ix amb l’eix X.

- El punt de tall Iy amb l’eix Y.

b) Passa aquests punts a un sistema de coordenades X-Y i amb la seva ajuda representa

gràficament la funció.

a) 32)( xxf b) 53)( xxf c) 45)( xxf

d) 3)( xxf e) xxf 4)( g) 232)( xxf

h) 4222)( xxf i) xxf 352)( j) 422)( xxf

k) 335)( xxf

Page 81: Funcions - Toomates · Operacions amb funcions. Composició de funcions. Funció inversa. 2 Les funcions de primer grau. → 3 Les funcions de segon grau. → 4 Les funcions polinòmiques.

7.5 Models amb arrels quadrades.

7.5.1

L'arrel quadrada com funció parametritzadora de la circumferència.

a) Dedueix, mitjançant el teorema de Pitàgores, l’equació de la circumferència amb

centre l’origen (0,0).

b) Dedueix de l’equació anterior les dues funcions )(1 xf i )(2 xf per a parametritzar

aquesta circumferència.

c) Aplica les dues funcions anteriors per a representar les circumferències següents:

c1: amb centre l’origen i radi 2

x -2 -1.5 -1 -0.5 0 0.5 1 1.5 2

)(1 xf

)(2 xf

c2: amb centre l’origen i radi 3

x -3 -2.5 -2 -1.5 -1 -0.5 0 0.5 1 1.5 2 2.5 3

)(1 xf

)(2 xf

Page 82: Funcions - Toomates · Operacions amb funcions. Composició de funcions. Funció inversa. 2 Les funcions de primer grau. → 3 Les funcions de segon grau. → 4 Les funcions polinòmiques.
Page 83: Funcions - Toomates · Operacions amb funcions. Composició de funcions. Funció inversa. 2 Les funcions de primer grau. → 3 Les funcions de segon grau. → 4 Les funcions polinòmiques.

7.6 Transformacions de funcions radicals

7.6.1

Transformació d'arrels cúbiques. Representa les següents funcions:

a) 3)( 31 xxf b) 2)( 3

2 xxf c) 5)( 33 xxf

Page 84: Funcions - Toomates · Operacions amb funcions. Composició de funcions. Funció inversa. 2 Les funcions de primer grau. → 3 Les funcions de segon grau. → 4 Les funcions polinòmiques.

7.6.2

Representa les següents funcions:

a) 31 4)( xxf b) 3

2 2)( xxf c) 33 5)( xxf

Page 85: Funcions - Toomates · Operacions amb funcions. Composició de funcions. Funció inversa. 2 Les funcions de primer grau. → 3 Les funcions de segon grau. → 4 Les funcions polinòmiques.

7.6.3

Representa les següents funcions:

a) 24)( 31 xxf b) 34)( 3

2 xxf

c) 12)( 33 xxf d) 23)( 3

3 xxf

Page 86: Funcions - Toomates · Operacions amb funcions. Composició de funcions. Funció inversa. 2 Les funcions de primer grau. → 3 Les funcions de segon grau. → 4 Les funcions polinòmiques.

7.6.4

Representa les següents funcions:

a) 31 )( xxf b) 3

2 )( xxf

c) 3)( 33 xxf d) 34)( 3

3 xxf

Page 87: Funcions - Toomates · Operacions amb funcions. Composició de funcions. Funció inversa. 2 Les funcions de primer grau. → 3 Les funcions de segon grau. → 4 Les funcions polinòmiques.

7.6.5

Associa cada funció amb la seva gràfica:

a) 22)( xxf b) 22)( xxf

c) 22)( xxf d) 22)( xxf

7.6.6

Donada la gràfica de la funció arrel quadrada:

Determina les expressions algèbriques associades a les següents funcions:

a) b) c)

Page 88: Funcions - Toomates · Operacions amb funcions. Composició de funcions. Funció inversa. 2 Les funcions de primer grau. → 3 Les funcions de segon grau. → 4 Les funcions polinòmiques.

El coneixement del comportament de les funcions pot ser la clau per resoldre problemes

d'equacions semblen molt complicats, com en el següent exemple:

Problema.

Donats els nombres reals 0,, cba , resol l'equació:

cxabxcaxbaxccxbbxa

Romania 1974 Solució: PA #4.1

Page 89: Funcions - Toomates · Operacions amb funcions. Composició de funcions. Funció inversa. 2 Les funcions de primer grau. → 3 Les funcions de segon grau. → 4 Les funcions polinòmiques.

7.10 Repàs d’equacions i funcions amb arrels quadrades (I).

7.10.1

Resol la següent equació: 52 x

7.10.2

Resol la següent equació: 11232 x

7.10.3

Resol la següent equació: 28 xx

7.10.4

Resol la següent equació: 47224 xx

7.10.5

Representa les següents funcions:

a) 2)(1 xxf

b) xxf )(2

c) 3)(3 xxf

Page 90: Funcions - Toomates · Operacions amb funcions. Composició de funcions. Funció inversa. 2 Les funcions de primer grau. → 3 Les funcions de segon grau. → 4 Les funcions polinòmiques.

7.10.6

Donada la funció 342)( xxf

a) Determina l’extrem P de la gràfica de la funció.

b) Determina el punt d’intersecció amb l’eix X.

c) Determina el punt d’intersecció amb l’eix Y.

d) Representa els punts anteriors i amb la seva ajuda representa gràficament la

funció.

7.10.7

a) Representa gràficament les següents funcions, i assenyala el seu punt de tall:

12)(1 xxf 32)(2 xxf

b) Determina algèbricament el punt de tall de les dues gràfiques anteriors.

Page 91: Funcions - Toomates · Operacions amb funcions. Composició de funcions. Funció inversa. 2 Les funcions de primer grau. → 3 Les funcions de segon grau. → 4 Les funcions polinòmiques.

7.10.8

a) Representa gràficament les següents funcions, i assenyala el seu punt de tall:

13)(1 xxf 42)(2 xxf

b) Determina algèbricament els possibles punts de tall de les dues gràfiques.

Page 92: Funcions - Toomates · Operacions amb funcions. Composició de funcions. Funció inversa. 2 Les funcions de primer grau. → 3 Les funcions de segon grau. → 4 Les funcions polinòmiques.

7.11 Repàs d’equacions i funcions amb arrels quadrades (II).

7.11.1

Resol les següents equacions:

a) 32 x b) 14812 x

c) 24 xx d) 2514 xx

7.11.2

Representa les següents funcions:

a) 3)(1 xxf b) xxf )(2 c) 4)(3 xxf

7.11.3

Donada la funció 583)( xxf

a) Determina l’extrem P de la gràfica de la funció.

b) Determina el punt d’intersecció amb l’eix X.

c) Determina el punt d’intersecció amb l’eix Y.

d) Representa els punts anteriors i amb la seva ajuda representa gràficament la funció.

Page 93: Funcions - Toomates · Operacions amb funcions. Composició de funcions. Funció inversa. 2 Les funcions de primer grau. → 3 Les funcions de segon grau. → 4 Les funcions polinòmiques.

7.11.4

a) Representa gràficament les següents funcions, i assenyala el seu punt de tall:

34)(1 xxf 12)(2 xxf

b) Determina algèbricament el punt de tall de les dues gràfiques anteriors.

7.11.5

Representa les següents funcions:

a) 31 )( xxf b) 34)( 3

2 xxf c) 33 )( xxf

7.11.6

Representa gràficament les següents funcions, i determina els seus punts de tall, de

forma gràfica i algèbrica:

21)( 31 xxf 1)(2 xxf

Page 94: Funcions - Toomates · Operacions amb funcions. Composició de funcions. Funció inversa. 2 Les funcions de primer grau. → 3 Les funcions de segon grau. → 4 Les funcions polinòmiques.

7.11.7

Representa gràficament una circumferència de radi 5 i centre en l’origen, mitjançant una

taula de valors de almenys 8 punts.

Page 95: Funcions - Toomates · Operacions amb funcions. Composició de funcions. Funció inversa. 2 Les funcions de primer grau. → 3 Les funcions de segon grau. → 4 Les funcions polinòmiques.

7.12 Repàs d’equacions i funcions amb arrels quadrades (III).

7.12.1

Resol les següents equacions:

a) 25 x b) 15227 x c) 79 xx

d) 41234 xx

7.12.2

Representa les següents funcions:

a) 1)(1 xxf b) xxf )(2 c) 2)(3 xxf

7.12.3

Donada la funció 263)( xxf

a) Determina l’extrem P de la gràfica de la funció.

b) Determina el punt d’intersecció amb l’eix X.

c) Determina el punt d’intersecció amb l’eix Y.

d) Representa els punts anteriors i amb la seva ajuda representa gràficament la funció.

Page 96: Funcions - Toomates · Operacions amb funcions. Composició de funcions. Funció inversa. 2 Les funcions de primer grau. → 3 Les funcions de segon grau. → 4 Les funcions polinòmiques.

7.12.4

Representa les següents funcions:

a) Representa gràficament les següents funcions, i assenyala el seu punt de tall:

21)(1 xxf 12)(2 xxf

b) Determina algèbricament el punt de tall de les dues gràfiques anteriors.

7.12.5

a) Representa gràficament les següents funcions, i assenyala el seu punt de tall:

22)(1 xxf 41)(2 xxf

b) Determina algèbricament els possibles punts de tall de les dues gràfiques.

Page 97: Funcions - Toomates · Operacions amb funcions. Composició de funcions. Funció inversa. 2 Les funcions de primer grau. → 3 Les funcions de segon grau. → 4 Les funcions polinòmiques.

8 Les funcions exponencials.

8.1 Característiques de les funcions polinòmiques.

Ara és un bon moment per repassar...

Exponencials i logaritmes (Tema 6 del Llibre d'Àlgebra)

Les equacions exponencials i logarítmiques (Tema 14 del Llibre d'Àlgebra)

Definició de la funció exponencial.

Definim la funció exponencial de base a (sempre suposant a > 0 ) com xaxf )(

Gràfica de la funció exponencial.

Si a>1 és creixent Si a<1 és decreixent.

La gràfica sempre és positiva i mai arriba a tocar l'eix de les x, tot i que s'hi aproxima

tant com es vulgui (això significa que l'eix de les x és una asímptota horitzontal de la

gràfica).

Propietats de les exponencials.

10 a

aa 1 yxyx aaa

x

x

xa

aa

11, en particular,

aa

11

xxx abba

Per a tot n>1, nn aa /1

En particular, fent 2.71828183 ea , tenim la funció exponencial de base e, o

simplement « exponencial » xexf )(

Per a calcular exponencials amb la calculadora científica (els models antics) farem

servir la tecla . Per exemple, per a calcular 67

Page 98: Funcions - Toomates · Operacions amb funcions. Composició de funcions. Funció inversa. 2 Les funcions de primer grau. → 3 Les funcions de segon grau. → 4 Les funcions polinòmiques.

8.1.1

Associa cada gràfica amb la seva expressió algèbrica.

i) x

xf1

)( ii) xxf 3)( iii) 4)( 2 xxf

iv) 3)( xxf v) xxxxf 6)( 23

A B C

D E

Page 99: Funcions - Toomates · Operacions amb funcions. Composició de funcions. Funció inversa. 2 Les funcions de primer grau. → 3 Les funcions de segon grau. → 4 Les funcions polinòmiques.

8.2 De l'expressió algèbrica a la gràfica.

8.2.1

Completa la taula i representa gràficament.

a) xxf 4)(

b) xxf 5.0)(

Page 100: Funcions - Toomates · Operacions amb funcions. Composició de funcions. Funció inversa. 2 Les funcions de primer grau. → 3 Les funcions de segon grau. → 4 Les funcions polinòmiques.

c) xxf 25.1)(

d) 2/2)( xxf

Page 101: Funcions - Toomates · Operacions amb funcions. Composició de funcions. Funció inversa. 2 Les funcions de primer grau. → 3 Les funcions de segon grau. → 4 Les funcions polinòmiques.

8.2.2

Representa gràficament les següents funcions.

a) xxf 24)( b) xxf 25)(

c)

x

xf

2

14)( d)

x

xf

2

12)(

Page 102: Funcions - Toomates · Operacions amb funcions. Composició de funcions. Funció inversa. 2 Les funcions de primer grau. → 3 Les funcions de segon grau. → 4 Les funcions polinòmiques.

e) 223)( 2 xxf f) 22

14)(

1

x

xf

Page 103: Funcions - Toomates · Operacions amb funcions. Composició de funcions. Funció inversa. 2 Les funcions de primer grau. → 3 Les funcions de segon grau. → 4 Les funcions polinòmiques.

8.2.3

Representa gràficament les següents funcions:

a) xxf 2)(1 b) xxf 3)(2 c) x

x

xf 5.02

1)(3

d) xxf 1.0)(4

Page 104: Funcions - Toomates · Operacions amb funcions. Composició de funcions. Funció inversa. 2 Les funcions de primer grau. → 3 Les funcions de segon grau. → 4 Les funcions polinòmiques.

8.3 De la gràfica a l'expressió algèbrica.

Exercici resolt.

Determina la funció exponencial xaxf )( que s'adapta a la següent gràfica:

xaxf )( , i sabem que passa pel punt )25,2( , per tant:

525)2(25 2 aaf

Per tant, la funció buscada és xxf 5)(

Exercici resolt.

Determina la funció exponencial xaxf )( que s'adapta a la següent gràfica:

xaxf )( , i sabem que passa pel punt )8/1,3( , per tant:

2

1

2

1

2

1)3(

8

1 3

3

3

3

aaaf

Per tant, la funció buscada és xxf )2/1()(

8.3.1

Determina la funció exponencial xaxf )( que s'adapta a cada gràfica:

Page 105: Funcions - Toomates · Operacions amb funcions. Composició de funcions. Funció inversa. 2 Les funcions de primer grau. → 3 Les funcions de segon grau. → 4 Les funcions polinòmiques.

8.3.2 Exercici resolt.

La següent gràfica mostra una funció del tipus xakxf )(

a) Determina els valors de a i k .

b) Avalua la funció per a 3x .

Solució:

a)

El punt )5,0( pertany a la gràfica de la funció, per tant 5)0( f

51)0(5 0 kkkakf

El punt )45,2( pertany a la gràfica de la funció, per tant 45)2( f 22 5455)2(45 aaf

Obtenim una equació de segon grau que hem de resoldre:

395

45545 222 aaaa

Les funcions exponencials sempre tenen la base positiva, per tant ens quedem amb la

solució positiva de l'equació: 3a

I arribem a xxf 35)(

Comprovem que, efectivament, aquesta funció satisfà les condicions de l'enunciat:

51535)0( 0 f

459535)2( 2 f

b)

Avaluem la funció:

13527535)3( 3 f

Page 106: Funcions - Toomates · Operacions amb funcions. Composició de funcions. Funció inversa. 2 Les funcions de primer grau. → 3 Les funcions de segon grau. → 4 Les funcions polinòmiques.

8.3.3

La següent gràfica mostra una funció del tipus xakxf )(

Determina els valors de a i k .

8.3.4

The diagram shows a sketch of the graph of xaby

The curve passes through the points A (0.5, 1) and B (2, 8)

The point C (-0.5, k) lies on the curve.

Find the value of k

Page 107: Funcions - Toomates · Operacions amb funcions. Composició de funcions. Funció inversa. 2 Les funcions de primer grau. → 3 Les funcions de segon grau. → 4 Les funcions polinòmiques.

8.3.5 Exercici resolt.

La següent gràfica mostra una funció del tipus exponencial xakxf )( . Determina

aquesta funció.

Solució:

Ens interessa treballar amb el valor de la funció 0x , per tant, farem una

transformació d'aquesta funció: Translladem la seva gràfica una unitat cap a l'esquerra.

Els punts ara són: )5,0( i )320,3( . Trobem la funció:

5)0(5

)(

0

kkakg

akxg x

464645

3205)3(320 3333 aaaaag

Per tant, la funció és xxg 45)(

I ara desfem la translació que hem fet, és a dir, ara translladem la gràfica una unitat cap

a la dreta. Recordem que "translladar una unitat cap a la dreta" és realitzar la

composició

)1()( xgxf

I, ara sí, la nostra funció és 145)1()( xxgxf

Si volem obtenir la seva expressió de la forma xakxf )( fem una mica d'àlgebra:

xxxxf 44

544545)( 11

Page 108: Funcions - Toomates · Operacions amb funcions. Composició de funcions. Funció inversa. 2 Les funcions de primer grau. → 3 Les funcions de segon grau. → 4 Les funcions polinòmiques.

8.4 Modelitzant amb funcions exponencials.

8.4.1

Creixement d’una població cel·lular (Aplicació de la funció exponencial).

Considerem que un bacteri es divideix cada 40 segons en dos nous bacteris, cadascun

dels quals es torna a dividir en dos més cada 40 segons i així successivament, és a dir,

que la població de bacteris es dobla cada 40 segons, sempre que les condicions del

cultiu bacteriològic no es modifiquin.

a) Completa la següent taula

Segons Bacteris

0 1

40

80

120

160

b) Representa mitjançant una fórmula matemàtica el nombre de bacteris que hi ha en

cada moment en funció del temps t en segons.

c) Quants bacteris hi haurà en el cultiu al cap de 10 minuts?

d) Quant de temps haurà de passar per a obtenir un cultiu de 1000000 de bacteris ?

e) Quant de temps haurà de passar per a obtenir un cultiu de 109 bacteris ?

8.4.2

La grandària d’un cert cultiu de bacteris es multiplica per 2 cada 30 minuts. Si suposem

que el cultiu té inicialment 5 milions de bacteris, quantes hores trigarà a tenir 320

milions de bacteris?

8.4.3

La grandària d’un cert cultiu de bacteris es multiplica per 2 cada 20 minuts. Si al cap de

3 hores el cultiu té 576 milions de bacteris, quants n’hi havia a l’instant inicial?

Page 109: Funcions - Toomates · Operacions amb funcions. Composició de funcions. Funció inversa. 2 Les funcions de primer grau. → 3 Les funcions de segon grau. → 4 Les funcions polinòmiques.

8.5 Transformacions de funcions exponencials.

Exercici resolt.

Utilitza la gràfica de la funció xxf 3)( per representar la gràfica de la funció 13)( xxg

Solució:

Estudiem la funció )(xg com una composició de funcions: )1()( xfxg

Aquesta composició de funcions es correspon, gràficament, a una translació horitzontal

d'1 unitat cap a l'esquerra.

Exercici resolt.

Utilitza la gràfica de la funció xxf 2)( per representar la gràfica de la funció

32)( xxg

Solució:

Estudiem la funció )(xg com una composició de funcions: 3)()( xfxg

Aquesta composició de funcions es correspon, gràficament, a una translació vertical de

3 unitats cap avall.

Page 110: Funcions - Toomates · Operacions amb funcions. Composició de funcions. Funció inversa. 2 Les funcions de primer grau. → 3 Les funcions de segon grau. → 4 Les funcions polinòmiques.

8.5.1

Per a cada funció, determina:

a) Punt de tall amb l’eix Y.

b) Possible punt de tall amb l’eix X.

c) Valors )1(f o )1(f

d) Asímptota horitzontal.

Amb els punts anteriors representa gràficament la funció.

Funció Model Transformació geomètrica

a) 32)( xxf

b) 559.0)( xxf

c) 13)( xxf

baxf x )(

d) xxf 32)(

e) xxf 25.0)(

f) xxf 2.03)(

xakxf )(

g) xxf 2)(

h) xxf 8.0)( xaxf )(

i) 423)( xxf

j) 24.0)( xxf

k) 534)( xxf

(Composició de dos o més)

Page 111: Funcions - Toomates · Operacions amb funcions. Composició de funcions. Funció inversa. 2 Les funcions de primer grau. → 3 Les funcions de segon grau. → 4 Les funcions polinòmiques.

8.6 Punts de tall amb funcions exponencials.

Exercici resolt.

Determinar el punt d’intersecció de les gràfiques de les funcions

352)( xxf 153)( xxg

Solució:

Igualem les dues fórmules:

045

1352

0153352153352)()(

x

z

xzxzxgxf

Fem el canvi de variable xp 5

041

32 p

p

Multiplicant per p obtenim una equació de segon grau:

2.58114p

-0.581139p

4

284

4

2444

22

)3(2424

03420432

2

22

p

pppp

Descartem la primera solució perquè una exponencial no pot ser negativa.

0.589169)5log(

)58114.2log()58114.2(log58114.25 5 xp x

2.16228352)589169.0( 589169.0 fy

El punt d’intersecció és ( 0.589 , 2.162 )

Page 112: Funcions - Toomates · Operacions amb funcions. Composició de funcions. Funció inversa. 2 Les funcions de primer grau. → 3 Les funcions de segon grau. → 4 Les funcions polinòmiques.

Exercici resolt.

Determina, aproximadament, els punts de tall entre les gràfiques de les funcions xexf )( i 2)( xxg

Solució:

Representem gràficament les dues funcions:

Observem que les dues gràfiques tenen un punt de tall: 5.0x aproximadament, i que

aquest serà el seu únic punt de tall possible (una és creixent i l'altra decreixent).

Si volem més precissió, podem tirar de calculadora:

022)()( xexexgxf xx

x 2 xe x

-0.5 0.149

-0.4 -0.108

-0.45 0.0183

-0.425 -0.045

Prenent sempre l'interval en el què canvia el signe anem ajustant més i més el valor de la

solució. Amb una computadora podem obtenir tants decimals com volguessim:

x = -0.44285440100238858314132799999933681971626212937347968471773...

8.6.1

Determinar, aproximadament, els punts de tall entre les gràfiques de les funcions xxf 2)( i xxg 4)(

Page 113: Funcions - Toomates · Operacions amb funcions. Composició de funcions. Funció inversa. 2 Les funcions de primer grau. → 3 Les funcions de segon grau. → 4 Les funcions polinòmiques.

8.10 Repàs de funcions exponencials.

8.10.1

Determina la funció exponencial xaxf )( si sabem que passa pel punt )3,1(P

8.10.2

Determina la funció exponencial xaxf )( si sabem que passa pel punt )3,7(P

8.10.3

Determina la funció exponencial xaxf )( si sabem que passa pel punt

)1.0,5.2(P . Podem esperar un valor a major o menor que 1?

8.10.4

Donada la funció xxf )7.0()( , determina les coordenades dels punts P, Q, R i S que

pertanyen a la seva gràfica:

on P = ( -2 , ? ) , Q = ( -1 , ? ) , R = ( 1.55 , ? ) i S = ( 3.9 , ? )

Page 114: Funcions - Toomates · Operacions amb funcions. Composició de funcions. Funció inversa. 2 Les funcions de primer grau. → 3 Les funcions de segon grau. → 4 Les funcions polinòmiques.

8.10.5

Donada la funció xxf )2.3()( , determina les coordenades dels punts P, Q, R i S que

pertanyen a la seva gràfica:

on P = ( ? , 2 ) , Q = ( ? , 0.25 ) , R = ( ? , 0.8 ) i S = ( ? , 0.1 )

8.10.6

Resol les següents equacions:

a) 0534 x b) 0643 x c) 08535 12 xx

d) 062 xx ee e) 022 2 xx

8.10.7

La població d’un país (mesurada en milions d’habitants) creix exponencialment de la

forma tetf 01.030)( , on la variable t representa els anys transcorreguts des de l’any

base 1980.

a) Completa la següent taula:

Any 1980 1981 1982 1985 1990 1995

Població

b) En quin any la població duplicarà la de 1980?

c) En quin any la població duplicarà la de 1990?

8.10.8

A partir de la gràfica de la funció xxf 2)( , representa gràficament les següents

funcions:

Page 115: Funcions - Toomates · Operacions amb funcions. Composició de funcions. Funció inversa. 2 Les funcions de primer grau. → 3 Les funcions de segon grau. → 4 Les funcions polinòmiques.

12)(1 xxf , 32)(2 xxf , 1

3 2)( xxf , 2

4 2)( xxf ,

xxf 2)(5 , 12)(6 xxf , i xxf 27 log)(

8.10.9

En el següent gràfic estan representades les funcions xxf 93.02.1)( i xxg 4.175.0)(

Associa cada funció amb la seva gràfica corresponent, i determina el punt A=(x,y) de

tall de les dues gràfiques.

8.10.10

Determina el punt A d’intersecció de les gràfiques de les funcions xxf 6.18)( i 12)( xxg . Assenyala prèviament la gràfica que correspon a cada funció.

Page 116: Funcions - Toomates · Operacions amb funcions. Composició de funcions. Funció inversa. 2 Les funcions de primer grau. → 3 Les funcions de segon grau. → 4 Les funcions polinòmiques.

8.10.11

Determina el punt A d’intersecció de les següents funcions: xxf )9.0(1200)( i xxg )13.1(750)( . Assenyala prèviament la gràfica que correspon a cada funció.

Problema D

Determina totes les solucions de l'equació:

xxxxx 1413121110

Solució: PA #6.5

Page 117: Funcions - Toomates · Operacions amb funcions. Composició de funcions. Funció inversa. 2 Les funcions de primer grau. → 3 Les funcions de segon grau. → 4 Les funcions polinòmiques.

9 Les funcions logarítmiques.

Ara és un bon moment per repassar...

Exponencials i logaritmes (Tema 6 del Llibre d'Àlgebra)

Les equacions exponencials i logarítmiques (Tema 14 del Llibre d'Àlgebra)

9.1 Característiques de les funcions logarítmiques.

Definició.

Definim el logaritme "de base a", i escriurem )(log)( xxf a com la funció inversa de

l'exponencial de base a: xaxg )( . y

a axxxfy )(log)(

Propietats dels logaritmes.

)(log)(log)(log yxxy aaa

)(log)(log)/(log yxyx aaa

)(log)(log xnx a

n

a , n

pp an

a

)(log)(log

Logaritmes especials.

Existeixen dos casos especials de logaritme:

- El logaritme en base 10 s'anomena logaritme decimal i es representa com a )log( x ,

sense cap tipus de base. El trobem a la calculadora científica a la tecla

- El logaritme en base e = 2.7182818... s'anomena logaritme natural o logaritme

neperià, i es pot representar com a )ln(x . El trobem a la calculadora científica a la tecla

Gràfica de les funcions logarítmiques.

Les funcions logarítmiques són inverses de les funcions exponencials. Recorda (Apartat

1.6) com són les gràfiques de les funcions inverses: Simètriques respecte la bisectriu xy

Page 118: Funcions - Toomates · Operacions amb funcions. Composició de funcions. Funció inversa. 2 Les funcions de primer grau. → 3 Les funcions de segon grau. → 4 Les funcions polinòmiques.

Domini de definició de les funcions logarítmiques.

Les funcions exponencials sempre donen valors positius, per tant els logaritmes només

estan definits per als valors positius.

Exercici resolt.

Determina el domini de la funció 24ln)( xxf

Solució:

04 2 xfDom 2204 2 xx 2,2fDom

Ara és un bon moment per repassar...

Les inequacions de segon grau. (Apartat 15.2 del Llibre d'Àlgebra)

Page 119: Funcions - Toomates · Operacions amb funcions. Composició de funcions. Funció inversa. 2 Les funcions de primer grau. → 3 Les funcions de segon grau. → 4 Les funcions polinòmiques.

9.2 De l'expressió algèbrica a la gràfica.

9.2.1

Representa gràficament les següents funcions:

a) )ln()(1 xxf b) )(log)( 22 xxf c) )(log)( 2.03 xxf

d) )log()(4 xxf e) )(log)( 6.05 xxf

Page 120: Funcions - Toomates · Operacions amb funcions. Composició de funcions. Funció inversa. 2 Les funcions de primer grau. → 3 Les funcions de segon grau. → 4 Les funcions polinòmiques.

Exercici resolt.

Determinar el punt d’intersecció de les gràfiques de les funcions

2)1(log)( 5 xxf 1)2(log)( 5 xxg

Solució:

Igualem les dues fórmules:

10.7805-

2

5091

2

)127(1411

2

)127(1411

0127

1252

5)2)(1(

3)2)(1(log

3)2(log)1(log

1)2(log2)1(log

)()(

2

2

3

5

55

55

x

xx

xx

xx

xx

xx

xx

xgxf

La primera solució no és acceptable perquè genera un logaritme d’un nombre negatiu.

La solució correcta és x = 11.7805

-0.4168882)17805.11(log)7805.11( 5 fy

El punt d’intersecció és ( 11.781 , -0.417 )

Page 121: Funcions - Toomates · Operacions amb funcions. Composició de funcions. Funció inversa. 2 Les funcions de primer grau. → 3 Les funcions de segon grau. → 4 Les funcions polinòmiques.

9.5 Transformacions de funcions logarítmiques.

Ara és un bon moment per repassar l'Apartat 1.4

Exercici resolt.

Representa gràficament la funció 2log)( 5 xxg

Solució:

En aquest cas veiem que la transformació realitzada és la següent: 2)()( xfxg , que

es correspon gràficament a una translació vertical de 2 unitats cap amunt:

Exercici resolt.

Representa gràficament la funció 3log)( 10 xxg

Solució:

En aquest cas veiem que la transformació realitzada és la següent: )3()( xfxg , que

es correspon gràficament a una translació horitzontal de 3 unitats cap a la dreta:

Page 122: Funcions - Toomates · Operacions amb funcions. Composició de funcions. Funció inversa. 2 Les funcions de primer grau. → 3 Les funcions de segon grau. → 4 Les funcions polinòmiques.

Exercici resolt.

Representa gràficament la funció

32log)( 4 xxg

Solució:

Primer pas: Representem la funció base )(log)( 4 xxf .

Sabem que el logaritme de 1 és sempre 0, i que el logaritme en base 4 de 4 és 1, per tant,

aquesta funció passa pels punts )0,1( i (4,1).

Sabem també que tot logaritme té asímptota vertical 0x .

Amb aquesta informació ja en tenim prou per representar-la:

Segon pas: Interpretem la funció 32log)( 4 xxg com 3)2()( xfxg , i per

tant la seva gràfica estarà situada 2 unitats a la dreta i 3 unitats cap amunt. Movem el

punts de referència i l'asímptota vertical.

Page 123: Funcions - Toomates · Operacions amb funcions. Composició de funcions. Funció inversa. 2 Les funcions de primer grau. → 3 Les funcions de segon grau. → 4 Les funcions polinòmiques.

Tercer pas: Unim els nous punts de referència, a mà alçada i sense por, seguint amb la

vista la funció base:

9.5.1

Representa gràficament les següents funcions, de forma aproximada i seguint les

indicacions de l'exercici resolt anterior. Determina el seu domini i el seu recorregut.

a) 51log)( 6 xxf b) 31log)( 5 xxf

Page 124: Funcions - Toomates · Operacions amb funcions. Composició de funcions. Funció inversa. 2 Les funcions de primer grau. → 3 Les funcions de segon grau. → 4 Les funcions polinòmiques.

c) 53log)( 6 xxf d) 31log)( 2 xxf

e) 41log)( 4 xxf f) 11log)( 5 xxf

g) 12log)( 4 xxf h) 12log)( 6 xxf

Page 125: Funcions - Toomates · Operacions amb funcions. Composició de funcions. Funció inversa. 2 Les funcions de primer grau. → 3 Les funcions de segon grau. → 4 Les funcions polinòmiques.

9.5.2

Associa cada expressió algèbrica amb la seva representació gràfica.

a) b)

c) d)

e) f)

i) 2log)( 3 xxf ii) xxf 3log)( iii) 2log)( 3 xxf

iv) )1(log)( 3 xxf v) )1(log)( 3 xxf )vi )(log)( 3 xxf

Page 126: Funcions - Toomates · Operacions amb funcions. Composició de funcions. Funció inversa. 2 Les funcions de primer grau. → 3 Les funcions de segon grau. → 4 Les funcions polinòmiques.

10 Les funcions trigonomètriques.

Trobareu el desenvolupament d'aquest tema a...

Les funcions trigonomètriques (Tema 3 del Llibre de Trigonometria)

Page 127: Funcions - Toomates · Operacions amb funcions. Composició de funcions. Funció inversa. 2 Les funcions de primer grau. → 3 Les funcions de segon grau. → 4 Les funcions polinòmiques.

11 Les funcions definides a trossos.

Definició de funció definida a trossos.

Les funcions definides a trossos són aquelles en les quals el valor de la funció s’obté de

diferent manera segons l’interval de la variable independent que s’està considerant.

Una funció definida a trossos té diferents fórmules o expressions algebraiques per a

distints intervals de la variable independent.

Per obtenir la gràfica d’una funció definida a trossos han de representar-se tots els seus

trams d’acord amb la funció present en cadascun; també cal prestar una atenció especial

al valor que adopta la funció en els extrems de cada interval.

Exercici resolt.

Determina l'expressió algèbrica associada a la següent gràfica:

A l'esquerra de 4x la funció és constant igual a 4:

4y si 4x

Entre 4x i 2x la gràfica es recta, i per tant la funció és de primer grau:

baxy

Amb pendent -1 bxy )1(

i val -2 quan 0x 20)1(2 bb

La funció és 2 xy

A la dreta de 2x , la gràfica es recta, i per tant la funció és de primer grau: baxy

Amb pendent 1 bxy 1

i val -4 quan 2x 6214 bb

La funció és 6 xy

Ara recollim tot en una única expressió:

2 xsi 6

24 si 2

4 si 4

)(

x

xx

x

xf

Page 128: Funcions - Toomates · Operacions amb funcions. Composició de funcions. Funció inversa. 2 Les funcions de primer grau. → 3 Les funcions de segon grau. → 4 Les funcions polinòmiques.

11.1.1

Relaciona cada funció amb la seva gràfica.

1.

1 si3

1 si4)(

xx

xxxf 2.

0 si4

0 si32)(

xx

xxxf 3.

0 si42

0 si4)(

xx

xxxf

4.

1 si5

1 si13)(

x

xxxf 5.

1 si2

1 si23)(

xx

xxxf 6.

1 si5

1 si13)(

x

xxxf

11.1.2

Determina l'expressió algèbrica associada a cada gràfica:

a) b)

Page 129: Funcions - Toomates · Operacions amb funcions. Composició de funcions. Funció inversa. 2 Les funcions de primer grau. → 3 Les funcions de segon grau. → 4 Les funcions polinòmiques.

11.1.3

Representa gràficament la següent funció:

3 si52

3 si3)(

xx

xxf

Determina )4(f )0(f )3(f

Page 130: Funcions - Toomates · Operacions amb funcions. Composició de funcions. Funció inversa. 2 Les funcions de primer grau. → 3 Les funcions de segon grau. → 4 Les funcions polinòmiques.

11.1.4

Representa gràficament la següent funció:

2 si12

2 si5)(

xx

xxxf

Determina )3(f )4(f )2(f

Page 131: Funcions - Toomates · Operacions amb funcions. Composició de funcions. Funció inversa. 2 Les funcions de primer grau. → 3 Les funcions de segon grau. → 4 Les funcions polinòmiques.

11.1.5

Representa gràficament les següents funcions.

a)

2 si4

2 si12)(

xx

xxxf b)

2 si42

22- si2

2 si4

)(

xx

xx

x

xf

c)

0 si4

04- si

4 si2

)(

2 xx

xx

x

xf

x

d)

-2 xsi)1(

2 si6)(

4x

xxf

e)

4 xsi4

4 si4

1

)(x

xxf f)

3 xsi

3

3 si2

)(

x

x

xf

x

g)

2 si2

24- si

4 si2

)(

xx

xx

xx

xf h)

1 si4

1 xsi1

1 si3

)(4

xx

x

xx

xf

Page 132: Funcions - Toomates · Operacions amb funcions. Composició de funcions. Funció inversa. 2 Les funcions de primer grau. → 3 Les funcions de segon grau. → 4 Les funcions polinòmiques.

11.3 La funció part entera.

Definim la part entera d'un nombre x com el nombre enter més gran que sigui igual o

menor que x. Es denota per

xxf )(

Per exemple: 21.2 , 457.4 , 88 , 22 , 44.3

La notació x va ser introduïda per Gauss al 1808. Nosaltres farem servir la notació

x , que va ser introduïda per Kenneth E. Iverson als anys 60 del segle passat.

Observa que equival a tatxar la part decimal només si el nombre es positiu. Si el

nombre és negatiu no és veritat:

Per exemple: 715.7 , 815.7

Gràfica de la funció part entera.

b

La funció "part entera" apareix sovint a les competicions matemàtiques:

Vegeu Tema 8 del llibre "Álgebra Problem Solving"

La funció "part entera" s'utilitza molt en aritmètica:

Vegeu Tema 13 del llibre "Teoría de números"

Page 133: Funcions - Toomates · Operacions amb funcions. Composició de funcions. Funció inversa. 2 Les funcions de primer grau. → 3 Les funcions de segon grau. → 4 Les funcions polinòmiques.

Problema. M

Determina (si és que existeixen) polinomis )(xf , )(xg y )(xh que compleixin:

022

0123

11

)()()(

xsix

xsix

xsi

xhxgxf

Putnam 1999 Solució: PA #7.3

Page 134: Funcions - Toomates · Operacions amb funcions. Composició de funcions. Funció inversa. 2 Les funcions de primer grau. → 3 Les funcions de segon grau. → 4 Les funcions polinòmiques.

12 La funció valor absolut. Definició de valor absolut.

El valor absolut d’un nombre real, |x|, coincideix amb el nombre si aquest és positiu o

nul, o bé amb el seu oposat, −x, si és negatiu; així doncs, pot considerar-se com una

funció definida a trossos, ja que aquesta operació exigeix canviar el signe del seu

argument:

0 si

0 si)(

xx

xxxxf

Gràfica del valor absolut.

La seva gràfica és:

La gràfica del valor absolut té una propietat molt important: Ës una funció que no es del

tot suau: Punxa en x=0, (passa la mà i notaràs com se't clava).

Propietats del valor absolut:.

a) 0a

b) aa

c) baba (Desigualtat triangular)

d) baba

e) b

a

b

a

Page 135: Funcions - Toomates · Operacions amb funcions. Composició de funcions. Funció inversa. 2 Les funcions de primer grau. → 3 Les funcions de segon grau. → 4 Les funcions polinòmiques.

12.2 Transformacions de la funció valor absolut.

12.2.1

Donada la gràfica de la funció valor absolut:

Determina les expressions algèbriques associades a les següents funcions:

a) b) c)

12.2.2

Representa gràficament les següents funcions:

a) 2)( xxf b) 2)( xxf

c) 23)( xxf d) 1)( 2 xxf

e) 34)( 2 xxxf f) )7)(2)(3()( xxxxf

g) xxxf )( h) x

xxf )( (“funció signe”)

i) )sin()( xxf j) 2

1)sin()( xxf

k) )ln()( xxf l) xxf )(

Page 136: Funcions - Toomates · Operacions amb funcions. Composició de funcions. Funció inversa. 2 Les funcions de primer grau. → 3 Les funcions de segon grau. → 4 Les funcions polinòmiques.

Problema resolt. Una funció peculiar.

La funció 168143)( xxxxxf

té un comportament molt sorprenent, propi d’una funció definida a trossos: és constant

igual a 1 entre x=5 i x=10:

Normalment pensem que una funció definida “algèbricament” (amb polinomis i arrels)

que sigui constant al llarg de tot un interval només pot ser la pròpia funció constant.

Aquesta funció podria ser un bon antiexemple. Ara bé, estudiant més detingudament

aquesta funció podem observar que apareixen valors absoluts.

El primer terme 143 xx en realitat és l’arrel quadrada del quadrat de

12 x , que és una forma “algèbrica” d’expressar un valor absolut, i el valor absolut

sí que és, en realitat, una funció definida a trossos:

521

5121212143

2

xsix

xsixxxxx

xxxx 51412012

De la mateixa manera, el segon terme es pot escriure com l’arrel quadrada del quadrat

de 13 x , que és una forma “algèbrica” d’expressar un valor absolut:

1031

10131313168

2

xsix

xsixxxxx

Per tant,

1312168143)( xxxxxxxf

Ara es veu més fàcilment que entre 5 i 10 aquesta funció “cancel·la” el terme 1x , i

per tant és constant:

11321)(105 xxxfx

En realitat, qualsevol funció de la forma )()()( xgbxgaxf tindrà aquest

peculiar comportament.

Page 137: Funcions - Toomates · Operacions amb funcions. Composició de funcions. Funció inversa. 2 Les funcions de primer grau. → 3 Les funcions de segon grau. → 4 Les funcions polinòmiques.

Atenció amb el tatxar quadrats i arrels!

xx 2 xx 2

xx 2 amb domini , xx 2 amb domini ,0

12.2.3

Donades les funcions 24)( xxf i xxg )( ,

a) Determina gf i el seu domini de definició.

b) Determina fg i el seu domini de definició.

c) Representa gràficament les dues funcions anteriors.

Page 138: Funcions - Toomates · Operacions amb funcions. Composició de funcions. Funció inversa. 2 Les funcions de primer grau. → 3 Les funcions de segon grau. → 4 Les funcions polinòmiques.

Solucions.

1.1.1 a) )6,6( b) )4,3( c) 2 i 4 d) -2 e) 2)4( f f) x = -3 , x = 5

1.1.2 a) 1 , -1, 3 , 4 b) 3,3Dom , 4,1Im c) 1.2 i -1.4 (aprox.)

d) -1 e) -3, 2 , 4 f) mín= ( )1,0( , màx=(3,4)

1.1.3 a) 3, 2, -2, 0 b) 4,4Dom , 3,2Im c) -1 i 1.8 (aprox.)

d) -2 e) mín= ( )2,0( , màx=(-4,3)

1.1.4 a) 3, -1 b) 4,4Dom , 3,2Im

c) 3,4Dom , 4,5.1Im (aprox.)

d) No n'hi ha

e) ( -2 , 1 ) , ( 2, 2)

1.1.5 a) 6 , 3 , 4 , 6 b) 8,1Dom , 2.6,2.1Im (aprox.)

c) ( 2 , 4 ) , ( 5 , 5) , ( 7 , 3 )

1.2.1

a) 523)4()4()4)(( gfgf

b) 010)0()0()0( fgfg

c) no definit, perquè )1(g no està definit.

d) 01

0

)0(

)0()0(

f

g

f

g

e) 0

1

)0(

)0()0(

g

f

g

f no definit.

1.2.2 a) 1129)4()4()4)(( gfgf

b) no està definit perquè )2(g no està definit.

c) 313)1()1()1)(( gfgf

d) no definit perquè 0)0( g

1.3.1 a) 3,10)3)(1(0340)( 2 xxxxxxxgf

b) 4082053205)( 222 xxxxxxxfg

1.3.2 4135239)52(09)52(0 22 xxxxxgf

1.6.1

xxx

fxgfxgf

8

3

83

3

8))(()(

xx

xgxfgxfg

3

8)83(83))(()(

Son funcions inverses una d'altra.

2.2.1

Page 139: Funcions - Toomates · Operacions amb funcions. Composició de funcions. Funció inversa. 2 Les funcions de primer grau. → 3 Les funcions de segon grau. → 4 Les funcions polinòmiques.

3.2.4 a) a) (0, 1) b) ( 0.27 , 0 ) i ( 3.73 , 0 ) c) (2,-3) d) x = 2

b) a) (0, -3) b) ( -2.58 , 0 ) i ( 0.58 , 0 ) c) (-1,-5) d) x = -1

c) a) (0, 1) b) ( -5.19 , 0 ) i ( 0.19 , 0 ) c) ( -2.5, 7.25) d) x = -2.5

d) a) (0, -10) b) ( -5 , 0 ) i ( 2 , 0 ) c) ( -1.5 , -12.25) d) x = -1.5

Page 140: Funcions - Toomates · Operacions amb funcions. Composició de funcions. Funció inversa. 2 Les funcions de primer grau. → 3 Les funcions de segon grau. → 4 Les funcions polinòmiques.

2.4.1 a) (-2, -11) b) ( 3 , 7 ) c) (-1,-1) d) )9/7,9/5(

3.3.1

La gràfica de la funció està oberta cap amunt.

L'eix de simetria és la recta x=-3

El seu vèrtex és el punt ( -3 , 1 )

El punt de tall amb l'eix Y és y=10

No té cap punt de tall amb l'eix X.

3.3.2 a f , b g , c j, d f

3.3.3 a) (3,1) b) c) (-1,5) d) e) (2,5) f) g) (-1,9)

3.3.4 a) Mínim: (2,-13) b) c) Màxim (1,1) d) e) mínim (1/2, -5/4)

3.3.5

a) a) 2)1()( 2 xxf

b) V=(1,2), Punts de tall eix X :Cap. Punt tall eix Y: 3

c)

d) IR, ,2

b) a) 9)3()( 2 xxf

b) V=(3,-9), Punts de tall eix X: 0, 6. Punt tall eix Y: 0

Page 141: Funcions - Toomates · Operacions amb funcions. Composició de funcions. Funció inversa. 2 Les funcions de primer grau. → 3 Les funcions de segon grau. → 4 Les funcions polinòmiques.

c)

d) IR, ,9

c) a) 3)1(3)( 2 xxf

b) V=(-1,-3), Punts de tall eix X: -2, 0. Punt de tall eix Y: 0

c)

d) IR, ,3

d) a) 13)3()( 2 xxf

b) V=(3,13). Punts de tall amb l'eix X: 133 . Punt de tall eix Y: 4

c)

d) IR, 13,

e) a) 1)1(2)( 2 xxf

b) V=(-1,1). Punts de tall eix X: Cap. Punt de tall eix Y: 3

c)

d) IR, ,1

f) a) 10)2/3(4)( 2 xxf

Page 142: Funcions - Toomates · Operacions amb funcions. Composició de funcions. Funció inversa. 2 Les funcions de primer grau. → 3 Les funcions de segon grau. → 4 Les funcions polinòmiques.

b) V=(-3/2,10). Punts de tall eix X: 2/102/3 . Punt de tall eix Y: 1

c)

d) IR, 10,

3.3.6 a) 22)( xxf b) 2)2(3)( xxf c) 2)3()( xxf

d) 1)( 2 xxf e) 5)2(2)( 2 xxf f) 439

4)(

2 xxf

3.3.7 a) xxxf 4)( 2 )4,2(4)2(4)( 22 vèrtexxxxxf

b) 22)( 2 xxxf )1,1(1)1()( 2 vèrtexxxf

c) 2106)( xxxf )1,3(1)3()( 2 vèrtexxxf

d) 238)( xxxf )4/41,2/3(4/41)2/3()( 2 vèrtexxxf

e) 982)( 2 xxxf )1,2(1)2(2)( 2 vèrtexxxf

3.4.1 a) 52)( 2 xxxf b) 34)( 2 xxxf c) 153)( 2 xxxf

3.5.1 a) ( 2 , 7 ) , ( -3 , -8 ) b) ( -1 ,4 ) , ( -4, -5) c) ( -4 , 5 ) , ( -2 , 1 )

3.5.2 ( -5 , -3 )

3.5.3 (0,-2), (4,2)

4.1.1 b i c

4.1.2 C

4.1.3 a) 4 b) 1/24 c) 16 d) 8

4.3.1 a) )2)(3)(5(2 xxx b) )4)(1)(1(3 xxx c) )3()1)(4(2 2 xxx

4.3.2

a)

Punt de tall amb l’eix Y: ( 0 , 9 )

Page 143: Funcions - Toomates · Operacions amb funcions. Composició de funcions. Funció inversa. 2 Les funcions de primer grau. → 3 Les funcions de segon grau. → 4 Les funcions polinòmiques.

Punts de tall amb l’eix X: -1, 1 i 3

-4 -2 2 4

-20

20

40

b)

Punt de tall amb l’eix Y: ( 0 , 18 )

Punts de tall amb l’eix X: -3, -1 i 3

-6 -4 -2 2 4 6

-75

-50

-25

25

50

75

100

c)

Punt de tall amb l’eix Y: ( 0 , 60 )

Punts de tall amb l’eix X: -5, -3, 1, 4

-6 -4 -2 2 4 6

-100

100

200

300

4.4.1

a) 8822)2)(1)(2(2)( 23 xxxxxxxf

b) 99)3)(3)(1()( 23 xxxxxxxf

c) 24161442)2)(3)(2)(1(2)( 234 xxxxxxxxxf

d) 45950105)5)(3)(3)(1)(1()( 2345 xxxxxxxxxxxf

4.4.2 i) C ii) A iii) D iv) B

4.4.3 a) 3/1a b) -1 , 3

c)

Page 144: Funcions - Toomates · Operacions amb funcions. Composició de funcions. Funció inversa. 2 Les funcions de primer grau. → 3 Les funcions de segon grau. → 4 Les funcions polinòmiques.
Page 145: Funcions - Toomates · Operacions amb funcions. Composició de funcions. Funció inversa. 2 Les funcions de primer grau. → 3 Les funcions de segon grau. → 4 Les funcions polinòmiques.

4.5.1

4.5.2

Page 146: Funcions - Toomates · Operacions amb funcions. Composició de funcions. Funció inversa. 2 Les funcions de primer grau. → 3 Les funcions de segon grau. → 4 Les funcions polinòmiques.

4.10.1 Punt de tall amb l’eix Y: (0, -4) Punt de tall amb l’eix X: 0,3/4

4.10.2 53)( xxf

4.10.3 a) C= ( 0 , 4 ) b) A= ( 1 , 0 ) i B= ( 4 , 0 ) c) D = (2.5 , -2.25)

d) la recta x = 2.5

e)

4.10.4 243)( 2 xxxf

4.10.5 A = (-0.436 , 3.309 ) B = ( 3.4364 , -8.309 )

4.10.6 )4)(1()2()( 2 xxxxf

4.10.7 246153)4)(1)(2(3)( 23 xxxxxxxf

4.10.8 a) Exemples: 1)( 2 xxf , 1)( 4 xxf

b) Exemples: 3)4()( xxf , 4)4()( xxf

4.10.9 a = 5 b = 4

4.11.1 Punt de tall amb l’eix Y: ( 0 , 1 ) Punt de tall amb l’eix X: ( 1/2 , 0 )

4.11.2 43)( xxf

4.11.3

a) A=(0,1)

b) B1 = ( 0.29 , 0 ) B2 = ( 1.71 , 0 )

c) V=(1,-1)

d) x = 1

e)

4.11.4 12)( 2 xxxf

4.11.5 A = ( -0.37 , -3.1 ) , B = ( 1.37 , 2.1 )

4.11.6 )1()2)(3()( 2 xxxxf

Page 147: Funcions - Toomates · Operacions amb funcions. Composició de funcions. Funció inversa. 2 Les funcions de primer grau. → 3 Les funcions de segon grau. → 4 Les funcions polinòmiques.

4.11.7 )3)(2)(1(2)( xxxxf

Punt de tall amb l’eix Y: ( 0 , 12 )

Punts de tall amb l’eix X: (1, 0) , (2, 0) i (-3,0)

4.11.8 xxxxf 1222)( 23

4.11.9 5a

4.11.10 -2 , 1 i 3

4.12.1 )1)(3)(5( xxx

4.12.2 -3, 1, 3, 5

4.12.3 306)2)(5)(3( 23 xxxxxx

4.12.4 903183)2)(5)(3(3 23 xxxxxx

4.12.5 22 )3()2()( xxxf (exemple)

4.12.6 )4)(1)(3()( xxxxxf

4.12.7 k = 3

4.12.8 k = 2, k = -1

4.12.9 a = -3 i b = 4

4.13.1 2.501

4.13.2 V = ( 0.333 , -0.666 )

4.13.3 A=(-2.303 , 5.908 ) B=(1.303 , -4.908 )

4.13.4 Eix Y: -48 , Eix X: { -2 , 2 , -4 }

4.13.5 f(x)=-1x2 + 3x + 2

4.13.6 P1=(-5,-22), P2=(-4,-14), P3=(2,-8)

4.13.7 241893)( 23 xxxxf

7.3.1

a)

,

2

3 b) ,44, c) ,35,

d) ,0 e) ,23, f) ),3()2,4[

g) ,32, h) ,1 i) ...5,43,2,0

7.3.2 a) x=10 b) x= 10 c) x=4

1

7.3.3 3,2Dom

7.4.1

a) P = (-2,-3), Ix = 7, Iy = -1.586

b) P = (-3,-5), Ix = 22, Iy = -3.268

Page 148: Funcions - Toomates · Operacions amb funcions. Composició de funcions. Funció inversa. 2 Les funcions de primer grau. → 3 Les funcions de segon grau. → 4 Les funcions polinòmiques.

c) P = (-5,-4), Ix = 11, Iy = -1.764

d) P = (0,-3), Ix = -9, Iy = -3

e) P = (4,0), Ix = 4, Iy = 2

f) P = (-2,2), Ix = -1.556, Iy = -2.243

Page 149: Funcions - Toomates · Operacions amb funcions. Composició de funcions. Funció inversa. 2 Les funcions de primer grau. → 3 Les funcions de segon grau. → 4 Les funcions polinòmiques.

g) P = (1,-4), Ix = -1, Iy = -1.172

h) P = (5/3,2), Ix = 0.333, Iy = -0.236

i) P = (-2,0), Ix = -2, Iy = -4

j) P = (5/3,-3), Ix = -1.333, Iy = -0.764

Page 150: Funcions - Toomates · Operacions amb funcions. Composició de funcions. Funció inversa. 2 Les funcions de primer grau. → 3 Les funcions de segon grau. → 4 Les funcions polinòmiques.

7.5.1 a) 222 Ryx

b) 22

1 )( xRxf 22

2 )( xRxf

c)

7.6.1

7.6.2

Page 151: Funcions - Toomates · Operacions amb funcions. Composició de funcions. Funció inversa. 2 Les funcions de primer grau. → 3 Les funcions de segon grau. → 4 Les funcions polinòmiques.

7.6.3

7.6.4

7.6.5 a) D b) B c) A d) C

7.6.6 a) 11)( xxf b) 1)( xxf c) 1)( xxf

7.10.1 x = 23

7.10.2 x = 7

7.10.3 x = 1

7.10.4 x = 21

7.10.5

Page 152: Funcions - Toomates · Operacions amb funcions. Composició de funcions. Funció inversa. 2 Les funcions de primer grau. → 3 Les funcions de segon grau. → 4 Les funcions polinòmiques.

7.10.6 a) (-2 , -3 ) b) ( 2.5 , 0 ) c) (0 , -1 )

d)

7.10.7

a)

b)

A = ( -0.36 , 2.28 )

Page 153: Funcions - Toomates · Operacions amb funcions. Composició de funcions. Funció inversa. 2 Les funcions de primer grau. → 3 Les funcions de segon grau. → 4 Les funcions polinòmiques.

7.10.8

a)

b) A = ( -1 , 3 ) B = ( 2 , 2 )

7.11.1 a) x = 11 b) x = 10 c) x = 5 d) x = 21

7.11.2

7.11.3

a) (-8/3 , -5 ) b) ( 17/3 , 0 ) c) (0 , -2.17 )

d)

7.11.4

a)

Page 154: Funcions - Toomates · Operacions amb funcions. Composició de funcions. Funció inversa. 2 Les funcions de primer grau. → 3 Les funcions de segon grau. → 4 Les funcions polinòmiques.

b) ( 0 , -1 )

7.11.5

7.11.6

( 0 , 1 ) , ( 1 , 2 ) i ( 2, 3 )

7.11.7

7.12.1 a) x = 9 b) x = 18 c) x = -5 d) x = 12

7.12.2

Page 155: Funcions - Toomates · Operacions amb funcions. Composició de funcions. Funció inversa. 2 Les funcions de primer grau. → 3 Les funcions de segon grau. → 4 Les funcions polinòmiques.

7.12.3 a) (-2 , -2 ) b) ( -0.67 , 0 ) c) (0 , 0.45 )

d)

7.12.4 a)

b)

A = ( 1.25 , 3.5 )

Page 156: Funcions - Toomates · Operacions amb funcions. Composició de funcions. Funció inversa. 2 Les funcions de primer grau. → 3 Les funcions de segon grau. → 4 Les funcions polinòmiques.

7.12.5

a)

b) A = ( -0.91 , 3.71 ) B = ( 1.91 , 2.29 )

8.1.1 i) C ii) B iii) A iv) E v) B

8.2.2

a) b)

c) d)

e) f)

Page 157: Funcions - Toomates · Operacions amb funcions. Composició de funcions. Funció inversa. 2 Les funcions de primer grau. → 3 Les funcions de segon grau. → 4 Les funcions polinòmiques.

8.2.3

8.3.1 a) xxf 3)( b) d) xxf 4/1)(

8.3.3 4k , 5a

8.4.1 a) 1 2 4 8 16

b) 40/2)( ttf

c) 32768

d) 800 segons (20 generacions)

e) 1200 segons (30 generacions)

Page 158: Funcions - Toomates · Operacions amb funcions. Composició de funcions. Funció inversa. 2 Les funcions de primer grau. → 3 Les funcions de segon grau. → 4 Les funcions polinòmiques.

8.5.1

a) a) ( 0 , -2) b) (1.585 , 0 ) c) f(1) = –1 d) y = -3

b) a) ( 0 , -4 ) b) (-3.0503 , 0 ) c) f(1) = –4.41 f(-1) = -3.305 d) y = -5

´

c) a) ( 0 , 2 ) b) no té c) f(1) = 4 f(-1) = 4/3 d) y = 1

d) a) ( 0 , 2 ) b) no té c) f(1) = 6 f(-1) = 2/3 d) y = 0

Page 159: Funcions - Toomates · Operacions amb funcions. Composició de funcions. Funció inversa. 2 Les funcions de primer grau. → 3 Les funcions de segon grau. → 4 Les funcions polinòmiques.

e) a) ( 0 , 0.5 ) b) no té c) f(1) = 1 f(-1) = 1/4 d) y = 0

f) a) ( 0 , 3 ) b) no té c) f(1) = 0.6 f(-1) = 15 d) y = 0

g) a) ( 0 , -1 ) b) no té c) f(1) = -2 f(-1) = -1/2 d) y = 0

h) a) ( 0 , -1 ) b) no té c) f(1) = -0.8 f(-1) = -1.25 d) y = 0

i) a) ( 0 , -1 ) b) ( 0.415 , 0 ) c) f(1) = 2 f(-1) = -5/2 d) y = -4

Page 160: Funcions - Toomates · Operacions amb funcions. Composició de funcions. Funció inversa. 2 Les funcions de primer grau. → 3 Les funcions de segon grau. → 4 Les funcions polinòmiques.

j) a) ( 0 , -3 ) b) no té c) f(1) = -2.4 f(-1) = -4.5 d) y = -2

k) a) ( 0 , 1 ) b) ( 0.203 , 0) c) f(1) = -7 f(-1) = 11/3 d) y = 5

8.6.1

Representant gràficament les dues funcions observem que el seu punt de tall és

aproximadament 4.1x

Anem provant valors per a obtenir una millor aproximació:

x xx 42

1.5 0.32

1 -1

1.25 -0.37

1.4 0.03

1.3 -0.23

1.35 -0.10

Per tant, 35.1x ( ...1.38616698 x )

8.10.1 xxf 3)(

Page 161: Funcions - Toomates · Operacions amb funcions. Composició de funcions. Funció inversa. 2 Les funcions de primer grau. → 3 Les funcions de segon grau. → 4 Les funcions polinòmiques.

8.10.2 xxf )16993.1()(

8.10.3 xxf )00316228.0()(

8.10.4 P=(-2, 2.04082), Q=(-1, 1.42857), R=(1.55, 0.57531) i S=( 3.9, 0.248818)

8.10.5 P=(0.595922, 2), Q=(-1.19184, 0.25), R=(-0.191844, 0.8) i S=(-1.97961 , 0.1)

8.10.6 a) x=0.203114, b) x=0.5, c) x=-1 d) x= 0.693147 e) x=1

8.10.7

a)

Any 1980 1981 1982 1985 1990 1995

Població 30 30.3015 30.606 31.5381 33.1551 34.855

b) Passats 69.3147 anys (és a dir, en l’any 2050)

c) Passats 79.3147 anys (és a dir, en l’any 2060)

8.10.9 A=(1.14903 , 1.104 )

8.10.10 A=(0.310826, 2.48084)

8.10.11 A=( 2.06524, 965.342)

Page 162: Funcions - Toomates · Operacions amb funcions. Composició de funcions. Funció inversa. 2 Les funcions de primer grau. → 3 Les funcions de segon grau. → 4 Les funcions polinòmiques.

9.2.1

9.5.1

a) 51log)( 6 xxf b) 31log)( 5 xxf

Domini: 1x , Recorregut: IR. Domini: 1x , Recorregut: IR.

c) 53log)( 6 xxf d) 31log)( 2 xxf

Page 163: Funcions - Toomates · Operacions amb funcions. Composició de funcions. Funció inversa. 2 Les funcions de primer grau. → 3 Les funcions de segon grau. → 4 Les funcions polinòmiques.

Imatge: 3x , Recorregut: IR. Domini: 1x , Recorregut: IR

e) 41log)( 4 xxf f) 11log)( 5 xxf

Domini: 1x , Recorregut: IR Domini: 1x , Recorregut: IR

g) 12log)( 4 xxf h) 12log)( 6 xxf

Domini: 2x , Recorregut: IR Domini: 2x , Recorregut: IR.

9.5.2 i) c ii) f iii) d iv) e v) b vi) a

Page 164: Funcions - Toomates · Operacions amb funcions. Composició de funcions. Funció inversa. 2 Les funcions de primer grau. → 3 Les funcions de segon grau. → 4 Les funcions polinòmiques.

11.1.5

a) b)

c) d)

e) f)

Page 165: Funcions - Toomates · Operacions amb funcions. Composició de funcions. Funció inversa. 2 Les funcions de primer grau. → 3 Les funcions de segon grau. → 4 Les funcions polinòmiques.

g) h)

12.2.1 a) xxf 3)( b) 11)( xxf c) 1)( xxf

12.2.2

a)

b)

c)

Page 166: Funcions - Toomates · Operacions amb funcions. Composició de funcions. Funció inversa. 2 Les funcions de primer grau. → 3 Les funcions de segon grau. → 4 Les funcions polinòmiques.

d)

e)

f)

g)

Page 167: Funcions - Toomates · Operacions amb funcions. Composició de funcions. Funció inversa. 2 Les funcions de primer grau. → 3 Les funcions de segon grau. → 4 Les funcions polinòmiques.

h)

i)

j)

k)

Page 168: Funcions - Toomates · Operacions amb funcions. Composició de funcions. Funció inversa. 2 Les funcions de primer grau. → 3 Les funcions de segon grau. → 4 Les funcions polinòmiques.

l)

12.2.3

a)

0 def. no

0444

222

xsi

xsixxxgxgfxgf

Domini de definició: ,0

b) 24)( xxfxfgxfg

Domini de definició: 2,204 2 x

c)